0% found this document useful (0 votes)
2K views58 pages

Toeic Introductory Answer Key

193. A Ms. Ramirez says there were no drapes at the window, and Mr. Evans writes that he would like a room fitted with blackout drapes. (B) Only Ms. Ramirez mentions the fact that some older people find it difficult to climb the stairs. (C) Only Mr. Evans refers to the content of the course (slides of paintings). (D) Only Ms. Ramirez talks about the items stocked in the cafeteria.194. D The sentence I appreciate your kind words comes after Mr. Evans’ thanks for the e-mail. Ms. Khan must have sai

Uploaded by

Nguyễn Minh
Copyright
© © All Rights Reserved
We take content rights seriously. If you suspect this is your content, claim it here.
Available Formats
Download as PDF, TXT or read online on Scribd
0% found this document useful (0 votes)
2K views58 pages

Toeic Introductory Answer Key

193. A Ms. Ramirez says there were no drapes at the window, and Mr. Evans writes that he would like a room fitted with blackout drapes. (B) Only Ms. Ramirez mentions the fact that some older people find it difficult to climb the stairs. (C) Only Mr. Evans refers to the content of the course (slides of paintings). (D) Only Ms. Ramirez talks about the items stocked in the cafeteria.194. D The sentence I appreciate your kind words comes after Mr. Evans’ thanks for the e-mail. Ms. Khan must have sai

Uploaded by

Nguyễn Minh
Copyright
© © All Rights Reserved
We take content rights seriously. If you suspect this is your content, claim it here.
Available Formats
Download as PDF, TXT or read online on Scribd
You are on page 1/ 58

lOMoARcPSD|9770843

ANSWER KEY
LISTENING COMPREHENSION
PART 1: PHOTOS C. 1. wearing
2. looking
Photo 1 (page 7)
3. smiling
A. 1. (N) 7. (?) 4. shaking
2. (N) 8. (?) 5. standing
3. (?) 9. (N)
D. 1. (Y)
4. (Y) 10. (?)
2. Neither man is wearing glasses.
5. (?) 11. (N)
3. Both men are wearing white shirts.
6. (Y) 12. (N)
4. Neither man has a long beard.
B. 1. (Y) 7. (N) 5. Neither man is dressed casually.
2. (Y) 8. (N) 6. Y
3. (N) 9. (N) 7. Neither man is bald.
4. (N) 10. (Y) 8. Neither man has a handkerchief in his
5. (N) 11. (N) pocket.
6. (Y) 12. (N)
Photo 3 (page 13)
C. 1. taking
A. 1. (N) 7. (N)
2. giving
2. (?) 8. (N)
3. facing
3. (Y) 9. (?)
4. sitting
4. (N) 10. (N)
5. touching
5. (N) 11. (N)
D. 1. Neither man is wearing a jacket. 6. (?) 12. (N)
2. (Y)
B. 1. (Y) 7. (Y)
3. (Y)
2. (N) 8. (N)
Copyright © 2012 by Pearson Education, Inc. Permission granted to reproduce for classroom use.

4. The man on the right has a watch on


3. (N) 9. (N)
his left wrist.
4. (Y) 10. (?)
5. There are few people in the office.
5. (N) 11. (N)
6. Both men are wearing white shirts.
6. (N) 12. (?)
7. The box is small and may or may not
be heavy. C. 1. watching
8. The man on the right is wearing a vest. 2. constructing
3. holding
Photo 2 (page 10)
4. following
A. 1. (N) 7. (?) 5. walking
2. (?) 8. (N)
D. 1. (Y)
3. (Y) 9. (N)
2. (Y)
4. (N) 10. (N)
3. Both men are wearing shoes.
5. (N) 11. (N)
4. Both men are wearing gloves.
6. (?) 12. (N)
5. The man in front is wearing a
B. 1. (N) 7. (Y) uniform.
2. (Y) 8. (N) 6. The man behind is wearing dark-
3. (N) 9. (N) colored pants.
4. (Y) 10. (N) 7. Both men have light-colored hard hats.
5. (N) 11. (Y) 8. Both men are wearing long pants.
6. (N) 12. (N)

INTRODUCTORY COURSE ANSWER KEY: LISTENING COMPREHENSION 1

Downloaded by Minh Nguy?n V?n (nguyenvanminh07012003@gmail.com)


lOMoARcPSD|9770843

Photo 4 (page 16) D. 1. (Y)


2. (Y)
A. 1. (?) 7. (?)
3. The pitchers are different sizes.
2. (?) 8. (N)
4. The table is round.
3. (?) 9. (N)
5. The tablecloth is a light color.
4. (N) 10. (Y)
6. (Y)
5. (N) 11. (?)
7. (Y)
6. (N) 12. (N)
8. The plates are clean.
B. 1. (Y) 7. (Y)
Photo 6 (page 23)
2. (Y) 8. (N)
3. (Y) 9. (N) A. 1. (N) 7. (N)
4. (N) 10. (Y) 2. (Y) 8. (N)
5. (Y) 11. (N) 3. (Y) 9. (?)
6. (N) 12. (N) 4. (Y) 10. (Y)
5. (?) 11. (N)
C. 1. styling
6. (Y) 12. (N)
2. having
3. holding B. 1. (Y) 7. (N)
4. looking 2. (N) 8. (Y)
5. sitting 3. (Y) 9. (N)
4. (N) 10. (N)
D. 1. Both women are wearing dark-
5. (Y) 11. (Y)
colored clothing; one is wearing
6. (N) 12. (Y)
pants.
2. (Y) C. 1. made
3. (Y) 2. turned on
4. Both women have long hair. 3. washed
5. (Y) 4. placed
6. The woman on the left is wearing 5. opened
dark-colored pants. D. 1. (Y)
7. Neither of the women is wearing 2. The bed is made.
glasses. 3. The rug covers only a small part of
Copyright © 2012 by Pearson Education, Inc. Permission granted to reproduce for classroom use.

8. The woman sitting down has light the floor.


hair. 4. (Y)
Photo 5 (page 20) 5. (Y)
6. (Y)
A. 1. (N) 7. (Y)
7. The sofa can seat two people comfortably.
2. (Y) 8. (Y)
8. The curtains are one color.
3. (Y) 9. (Y)
4. (N) 10. (N) Photo 7 (page 26)
5. (N) 11. (N) A. 1. (Y) 7. (?)
6. (N) 12. (Y) 2. (Y) 8. (N)
B. 1. (Y) 7. (N) 3. (N) 9. (N)
2. (N) 8. (Y) 4. (Y) 10. (Y)
3. (N) 9. (N) 5. (?) 11. (Y)
4. (Y) 10. (Y) 6. (N) 12. (N)
5. (N) 11. (N) B. 1. (Y) 7. (Y)
6. (Y) 12. (Y) 2. (Y) 8. (N)
C. 1. folded 3. (N) 9. (N)
2. took off 4. (N) 10. (Y)
3. set 5. (N) 11. (N)
4. filled 6. (Y) 12. (Y)
5. placed

INTRODUCTORY COURSE ANSWER KEY: LISTENING COMPREHENSION 2

Downloaded by Minh Nguy?n V?n (nguyenvanminh07012003@gmail.com)


lOMoARcPSD|9770843

C. 1. leaning PART 2: QUESTION-RESPONSE


2. closed
Identifying Time (page 42)
3. attached
4. hanging 1. (B) 6. (C)
5. left 2. (A) 7. (A)
3. (A) 8. (B)
D. 1. (Y)
4. (C) 9. (C)
2. The road is a brick road.
5. (A) 10. (A)
3. There aren’t any people on the street.
4. The bicycles are adult-sized. Identifying People (page 44)
5. (Y) 1. (B) 6. (B)
6. The building is several stories high. 2. (B) 7. (A)
7. (Y) 3. (A) 8. (B)
8. (Y) 4. (B) 9. (C)
Photo 8 (page 29) 5. (C) 10. (A)
A. 1. (Y) 7. (?) Identifying an Opinion (page 46)
2. (N) 8. (Y) 1. (A) 6. (C)
3. (Y) 9. (Y) 2. (C) 7. (B)
4. (?) 10. (?) 3. (A) 8. (A)
5. (Y) 11. (Y) 4. (B) 9. (C)
6. (N) 12. (?) 5. (C) 10. (B)
B. 1. (Y) 7. (N) Identifying a Choice (page 48)
2. (Y) 8. (N)
3. (?) 9. (N) 1. (A) 6. (B)
4. (Y) 10. (N) 2. (A) 7. (C)
5. (Y) 11. (Y) 3. (C) 8. (A)
6. (N) 12. (Y) 4. (A) 9. (C)
5. (B) 10. (B)
C. 1. planted
2. built Identifying a Suggestion (page 50)
3. separates 1. (A) 6. (C)
Copyright © 2012 by Pearson Education, Inc. Permission granted to reproduce for classroom use.

4. moves 2. (C) 7. (B)


5. painted 3. (A) 8. (A)
D. 1. (Y) 4. (B) 9. (B)
2. The leaves are still on the trees. 5. (A) 10. (C)
3. (Y) Identifying a Reason (page 52)
4. The boat doesn’t have any sails.
1. (B) 6. (C)
5. The river is calm.
2. (A) 7. (A)
6. (Y)
3. (A) 8. (B)
7. There is only one boat on the river.
4. (C) 9. (A)
8. It’s a sunny day.
5. (B) 10. (C)
Strategy Practice (page 34)
Identifying a Location (page 54)
1. (D) 6. (D)
1. (B) 6. (C)
2. (A) 7. (B)
2. (A) 7. (A)
3. (B) 8. (C)
3. (B) 8. (B)
4. (C) 9. (A)
4. (B) 9. (B)
5. (A) 10. (B)
5. (A) 10. (C)

INTRODUCTORY COURSE ANSWER KEY: LISTENING COMPREHENSION 3

Downloaded by Minh Nguy?n V?n (nguyenvanminh07012003@gmail.com)


lOMoARcPSD|9770843

Strategy Practice (page 57) Identifying a Location (page 75)


1. (B) 11. (B) 1. (D) 6. (B)
2. (A) 12. (C) 2. (B) 7. (A)
3. (C) 13. (B) 3. (A) 8. (B)
4. (C) 14. (A) 4. (B) 9. (D)
5. (A) 15. (C) 5. (D) 10. (D)
6. (C) 16. (C)
7. (C) 17. (A) Identifying an Opinion (page 77)
8. (B) 18. (B) 1. (B) 6. (C)
9. (A) 19. (A) 2. (D) 7. (C)
10. (A) 20. (B) 3. (A) 8. (C)
4. (D) 9. (A)
PART 3: CONVERSATIONS 5. (A) 10. (B)
Identifying Time (page 61) Identifying Stress and Tone (page 80)
1. (A) 6. (A) 1. (B) 6. (B)
2. (A) 7. (D) 2. (B) 7. (D)
3. (B) 8. (D) 3. (C) 8. (C)
4. (A) 9. (C) 4. (A) 9. (C)
5. (B) 10. (B) 5. (B) 10. (A)

Identifying People (page 64) Strategy Practice (page 83)


1. (B) 6. (D) 1. (B) 9. (D)
2. (C) 7. (A) 2. (A) 10. (D)
3. (A) 8. (B) 3. (B) 11. (A)
4. (B) 9. (C) 4. (C) 12. (B)
5. (D) 10. (C) 5. (B) 13. (B)
6. (D) 14. (D)
Identifying Intent (page 67)
7. (B) 15. (D)
1. (C) 6. (B) 8. (B)
2. (C) 7. (D)
3. (A) 8. (C) PART 4: TALKS
Copyright © 2012 by Pearson Education, Inc. Permission granted to reproduce for classroom use.

4. (B) 9. (A) Identifying the Sequence (page 89)


5. (D) 10. (D)
1. (D) 6. (D)
Identifying the Topic (page 69) 2. (A) 7. (B)
1. (D) 6. (A) 3. (B) 8. (B)
2. (B) 7. (D) 4. (A) 9. (B)
3. (C) 8. (C) 5. (C) 10. (A)
4. (A) 9. (B) Identifying the Audience (page 92)
5. (C) 10. (B)
1. (C) 6. (A)
Identifying a Reason (page 72) 2. (A) 7. (D)
1. (C) 6. (A) 3. (B) 8. (C)
2. (A) 7. (B) 4. (B) 9. (A)
3. (C) 8. (D) 5. (D) 10. (B)
4. (B) 9. (C)
5. (D) 10. (C)

INTRODUCTORY COURSE ANSWER KEY: LISTENING COMPREHENSION 4

Downloaded by Minh Nguy?n V?n (nguyenvanminh07012003@gmail.com)


lOMoARcPSD|9770843

Identifying a Location (page 94)


1. (B) 6. (C)
2. (A) 7. (A)
3. (B) 8. (C)
4. (D) 9. (C)
5. (D) 10. (C)
Identifying the Topic (page 97)
1. (D) 6. (B)
2. (A) 7. (A)
3. (C) 8. (A)
4. (B) 9. (B)
5. (A) 10. (D)
Identifying a Request (page 99)
1. (B) 6. (D)
2. (C) 7. (A)
3. (C) 8. (A)
4. (A) 9. (B)
5. (C) 10. (D)
Strategy Practice (page 103)
1. (B) 9. (B)
2. (D) 10. (B)
3. (A) 11. (D)
4. (A) 12. (D)
5. (B) 13. (A)
6. (D) 14. (B)
7. (C) 15. (C)
8. (B)
Copyright © 2012 by Pearson Education, Inc. Permission granted to reproduce for classroom use.

INTRODUCTORY COURSE ANSWER KEY: LISTENING COMPREHENSION 5

Downloaded by Minh Nguy?n V?n (nguyenvanminh07012003@gmail.com)


lOMoARcPSD|9770843

ANSWER KEY
LISTENING COMPREHENSION
REVIEW
PART 1 (PAGE 105) the similar-sounding word writing for
waiting.
1. (A) The man is photocopying a document.
Choice (B) tries to confuse you by 5. (A) The patrons are eating a meal. Choice
providing incorrect details: He’s lifting (B) tries to confuse you by using the
the photocopier lid, not opening a drawer. related words waiter, serving, and
Choice (C) tries to confuse you by using customers. Choice (C) tries to confuse you
the related word paper. Choice (D) tries to by using the related words chefs and
confuse you by using the related word file cooking. Choice (D) tries to confuse you by
and provides incorrect details: The man’s using the related word diners.
hand is on the paper, but he’s not handing it 6. (D) A man is talking into a microphone.
over to anyone. Choice (A) confuses similar-sounding
2. (D) The girls are walking through the words talking and walking and microphone
leaves beside the wall. Choice (A) tries to and home. Choice (B) confuses similar-
confuse you by using the similar-sounding sounding words microphone and
word raking for walking. Choice (B) tries to microscope. Choice (C) correctly identifies
confuse you by using the similar-sounding the action, speaking, but confuses
word trees for leaves and provides incorrect microphone with telephone.
details: They’re walking alongside the trees, 7. (B) The road is lined with trees. Choice (A)
not climbing the trees. Choice (C) provides provides incorrect details: There are sand
incorrect details: They’re wading through dunes in the picture, not snow; the road is
the leaves, not the water. small, not a highway, and is not covered
Copyright © 2012 by Pearson Education, Inc. Permission granted to reproduce for classroom use.

3. (B) The motorbikes are parked in a lot. with snow. Choice (C) provides incorrect
Choice (A) provides incorrect details: details: There is only one person on a bike
There is no light in the picture, and the on the road. Choice (D) tries to confuse
cycles are parked, not stopped. Choice (C) you by using related words forest (many
tries to confuse you by using the similar- trees) and sea (sand).
sounding word bicycles for motorcycles. 8. (D) The woman is washing the dishes.
Choice (D) uses the word park in a Choice (A) provides incorrect details: She
different context: The cycles are parked, but is washing a water glass, not watering the
we do not see the driver and do not know plants. Choice (B) provides incorrect
if they’re in a park (recreational area). details: She is cleaning the dishes, not her
4. (A) We can assume the women on the clothes. Choice (C) incorrectly infers she is
sidewalk in front of the pedestrian taking a bath.
crossing are waiting to cross the street. 9. (B) The passengers are closing the
Choice (B) provides incorrect details: One overhead bins on the plane. Choice (A)
woman is carrying a handbag and one has a tries to confuse you by using the related
suitcase, but they’re not shopping for bags. word suitcases. Choice (C) tries to confuse
Choice (C) provides incorrect details: you by using the related word flight.
While there are cars in the picture, the Choice (D) provides incorrect details:
women are not getting into any of them. They’re shutting the overhead bins, not the
Choice (D) tries to confuse you by using door to the plane. All of the subjects are
words associated with air travel.

INTRODUCTORY COURSE ANSWER KEY: LISTENING COMPREHENSION REVIEW 6

Downloaded by Minh Nguy?n V?n (nguyenvanminh07012003@gmail.com)


lOMoARcPSD|9770843

10. (B) We can assume the man looking at the 20. (A) About thirty-eight hundred dollars
plans is the construction manager. Choice answers how much. Choice (B) associates
(A) tries to confuse you by using the figures with cost. Choice (C) repeats the
related word specifications. Choice (C) tries word project and answers when.
to confuse you by using the related word 21. (A) Let’s hope everybody gets off at the next
supervisor. Choice (D) tries to confuse you stop responds to the statement This bus is
by using the related word crew. really crowded. Choice (B) contradicts the
meaning of the statement. There are
PART 2 (PAGE 111) many people on the bus. Choice (C)
11. (B) At 3 o’clock answers when (what time). repeats the word really and confuses the
Choice (A) answers where. Choice (C) similar-sounding word cloudy for
answers how long. crowded.
12. (A) No, but I’ll call the airline answers the 22. (B) Not yet answers Are the board minutes
yes/no question and provides an action. ready. Choice (A) confuses similar sounds
Choice (B) repeats the word found. Choice distributed and distribution. Choice (C)
(C) confuses similar sounds light and confuses the board meeting and the board
flight. minutes.
13. (B) I’ll come get you responds to need to be 23. (A) Sounds good to me is an enthusiastic
picked up. Choice (A) repeats the words response to How about a long lunch break.
picked up. Choice (C) repeats the word Choice (B) confuses similar sounds earlier
Saturday. and early. Choice (C) repeats the phrase
14. (A) At my house answers where. Choice (B) lunch break and confuses over already and
repeats the words party and year. Choice finish early.
(C) repeats the word office and confuses 24. (C) A fifteen-minute walk answers how far.
similar sounds near and year. Choice (A) repeats the word center and
15. (C) Sanderson Accounting answers what’s confuses similar sounds inconvenient and
the name of. Choice (A) confuses similar convention. Choice (B) repeats the word
sounds accounts and accounting. Choice hotel and confuses similar sounds now
(B) repeats the word firm. and how.
Copyright © 2012 by Pearson Education, Inc. Permission granted to reproduce for classroom use.

16. (B) I think we’re lost responds to the 25. (B) You better hurry. The show’s about to
statement We’re getting close. Choice (A) start responds to the request Save my seat.
confuses the similar sounds of close and Choice (A) uses the associated word left
clothes. Choice (C) repeats forms of the for the direction right. However, in this
verb get/got. context right means very soon. Choice (C)
uses the associated word wrong for right.
17. (A) Not that I know of answers Haven’t Again, right has a different meaning in
they located the problem yet. Choice (B) this context.
confuses similar sounds location and
located. Choice (C) repeats the words 26. (B) No, I have to pick new dates provides a
have, problem, and yet. negative response with a reason it wasn’t
approved. Choice (A) repeats the word
18. (C) It’ll be held at this office answers Will vacation. Choice (C) repeats the word
the seminar be held here or at the main office. approve.
Choice (A) confuses meeting and seminar.
Choice (B) confuses hear and here. 27. (A) Yes, if we buy in quantity answers the
yes/no question Doesn’t our company get a
19. (B) The same as last year answers Who are special price. Choice (B) confuses company
the board members this year. Choice (A) (companionship) and company (business).
confuses bored and board. Choice (C) Choice (C) confuses similar sounds phone
confuses similar sounds aboard and board. and phones and associates office with
company.

INTRODUCTORY COURSE ANSWER KEY: LISTENING COMPREHENSION REVIEW 7

Downloaded by Minh Nguy?n V?n (nguyenvanminh07012003@gmail.com)


lOMoARcPSD|9770843

28. (C) Sorry, I have to work that day provides 37. (C) Do you want to eat somewhere else
a response by giving the reason that the answers Isn’t this the same place by
speaker cannot come. Choice (A) offering an alternative. Choice (A)
confuses similar sounds picked and picnic confuses similar sounds race and place
and repeats the word Sunday. Choice (B) and repeats the word Tuesday. Choice (B)
repeats the word picnic and confuses repeats the words same and lunch.
similar sounds fun and come. 38. (A) Put on a sweater is the advice given to
29. (A) The Hotel International answers where. someone who thinks It’s very cold in here.
Choice (B) repeats the words suggest and Choice (B) repeats the word very and uses
stay. Choice (C) confuses stay (n.) and the similar-sounding old for cold and the
stay (v.) and repeats the word Singapore. word She for a word that sounds like her:
here. Choice (C) contradicts the meaning
30. (B) Yes, they should arrive tomorrow
of the statement. If it is cold, you turn on
answers the yes/no question Have you
the heat, not off.
ordered the supplies. Choice (A) associates
staples and paper with supplies. Choice (C) 39. (C) He likes to be informed of events
repeats the word for and associates pay answers Why does Michael always watch the
with order. news. Choice (A) confuses the similar
sounds knew and news, and broke and
31. (A) Right after lunch answers when.
break, and confuses watch (n.) and watch
Choice (B) confuses When did you move
(v.). Choice (B) repeats the word always
and When can you move the copy machine.
and confuses similar sounds breakfast and
Choice (C) associates minutes with time
break.
and answers how long will it take.
40. (C) Yes, that’s the sale price you’re looking at
32. (C) No, I’m new answers the yes/no
answers Isn’t this suit already discounted.
question Are you our new committee
Choice (A) confuses the similar words
chairperson. Choice (A) confuses chairs
suits (v.) and suit (n.). Choice (B) confuses
and chairperson. Choice (B) repeats the
similar sounds discounts and discounted
word committee and answers Am I on the
and suit (v.) and suit (n.).
committee.
33. (C) It looks like you’ve lost some weight is an PART 3 (PAGE 112)
Copyright © 2012 by Pearson Education, Inc. Permission granted to reproduce for classroom use.

observation on the pants that are too big.


41. (B) The man says I have lots of e-mail to
Choice (A) uses similar-sounding dance
answer. Choice (A) confuses mailing a
for pants. Choice (B) uses the similar-
letter with e-mail. Choice (C) confuses
sounding France for pants and the
answering the phone with e-mail to answer.
synonym large for big.
Choice (D) repeats the word computer, but
34. (A) Talk to Mrs. Durfee in Human Resources the man is using the computer, not
answers how can I get more information. repairing it.
Choice (B) confuses car insurance and
42. (A) The woman says Let’s go eat. Choice
health insurance. Choice (C) repeats the
(B) associates coffee with cafeteria. Choice
word health.
(C) confuses go to bed with the similar-
35. (B) No, he didn’t attend answers the yes/no sounding go ahead. Choice (D) confuses
question The manager was present at the word seat with the similar-sounding
yesterday’s staff meeting. Choice (A) word eat.
repeats the phrase staff meeting. Choice
43. (D) The man says he’ll be in the cafeteria
(C) confuses present (n.) and present (v.).
in fifteen minutes. Choice (A) is not
36. (A) I thought I could handle it myself mentioned. Choice (B) confuses the teen
answers Why didn’t you tell by giving a of fifteen with the similar-sounding word
reason. Choices (B) and (C) are not ten. Choice (C) confuses five with fifteen.
appropriate responses.

INTRODUCTORY COURSE ANSWER KEY: LISTENING COMPREHENSION REVIEW 8

Downloaded by Minh Nguy?n V?n (nguyenvanminh07012003@gmail.com)


lOMoARcPSD|9770843

44. (C) The speakers mention overhead 52. (C) The woman says that she is planning
compartment, on board, fly, and flight, so for six guests. Choice (A) confuses two
they are on a plane. Choice (A) confuses with the similar-sounding word to.
apartment with the similar-sounding Choice (B) confuses four with the similar-
word compartment. Choice (B) associates sounding word for. Choice (D) confuses
office with laptop. Choice (D) associates ten with the similar-sounding word then.
hotel with nap and serve a meal. 53. (B) The speakers agree to go to the later
45. (A) The woman says I’ll take a nap. Choice presentation, which begins at 11:00.
(B) associates work with laptop. Choice (C) Choice (A) is the time of the earlier
associates cook with meal. Choice (D) is presentation. Choice (C) uses the word
not mentioned. one in a different context. Choice (D) is
the time their train will leave.
46. (D) The man says that he is hungry.
Choice (A) confuses bored with board. 54. (A) It is a cooking demonstration. Choice
Choice (B) is how the woman feels. (B) confuses books with the similar-
Choice (C) confuses angry with the sounding word cook. Choice (C) repeats a
similar-sounding word hungry. word mentioned in another part of the
conversation. Choice (D) confuses games
47. (B) The man is helping a woman who has
with the similar-sounding word train.
arrived for an appointment, so he is a
receptionist. Choice (A) uses the word 55. (B) The man suggests having lunch.
clock in a different context. Choice (C) is Choice (A) is what the woman suggests
not mentioned. Choice (D) confuses the doing. Choice (C) confuses watch with the
job of an usher with the man’s offer to similar-sounding word catch. Choice (D)
have a seat. confuses look with the similar-sounding
word cook.
48. (A) The woman says she has an
appointment at 1:00. Choice (B) confuses 56. (B) The manager said that the report is
four with the similar-sounding word for. due Tuesday. Choice (A) is the day the
Choice (C) confuses eight with the speakers will work on the report. Choice
similar-sounding word late. Choice (D) (C) is the day the speakers thought that
confuses nine with the similar-sounding the report was due. Choice (D) confuses
Copyright © 2012 by Pearson Education, Inc. Permission granted to reproduce for classroom use.

word kind. Thursday with the similar-sounding word


Tuesday.
49. (B) The woman accepts the man’s offer to
have a seat. Choice (A) confuses call with 57. (C) The woman calls it an expense report.
the similar-sounding word all. Choice (C) Choice (A) confuses news with new.
confuses ball with the similar-sounding Choice (B) confuses management with
word all. Choice (D) repeats the word manager. Choice (D) repeats the word
appointment, but the woman never says meeting in a different context.
she wants a new one. 58. (D) The man says that he is mad. Choice
50. (B) Thursday is the day the woman plans (A) confuses sad with the similar-
to invite people for dinner. Choice (A) sounding word mad. Choice (B) is not
confuses Tuesday with the similar- mentioned. Choice (C) confuses ill with
sounding word Thursday. Choices (C) and the similar-sounding word will.
(D) are days mentioned by the man. 59. (A) The speakers mention a resort,
51. (C) The woman plans to invite neighbors. restaurants, pool, beds, and fitness room, so
Choice (A) confuses business with the they are in a hotel. Choice (B) repeats the
similar-sounding word busy. Choice (B) is word restaurant. Choice (C) associates
mentioned by the man. Choice (D) beach with resort. Choice (D) associates
confuses sisters with the similar- apartment building with beds, pool, and
sounding words six guests. fitness room.

INTRODUCTORY COURSE ANSWER KEY: LISTENING COMPREHENSION REVIEW 9

Downloaded by Minh Nguy?n V?n (nguyenvanminh07012003@gmail.com)


lOMoARcPSD|9770843

60. (C) The woman says And best of all, Choice (D) is mentioned by the man as an
comfortable beds. Choices (A), (B), and (D) acceptable form of payment.
are things mentioned by the speakers but 69. (B) The woman is buying shoes. Choice
none is mentioned as the woman’s (A) confuses cards with credit cards.
favorite. Choice (C) confuses book with the similar-
61. (D) The man says he plans to spend the sounding word look. Choice (D) repeats
day in the fitness room. Choice (A) the word purse.
associates sleep with beds. Choice (B) 70. (C) The man says that the shoes are $75.
associates eat with restaurants. Choice (C) Choice (A) confuses $17.50 with the
associates swim with pool. similar-sounding number 75. Choice (B)
62. (B) The woman is shopping for envelopes is the amount of change the man gives
in a store. Choice (A) confuses waiting the woman. Choice (D) is the size of the
room with the related word wait. Choice bill the woman uses to pay for the shoes.
(C) associates post office with letter.
Choice (D) associates library with books PART 4 (PAGE 115)
by confusing books with the similar- 71. (C) The speaker is the captain and
sounding word box. mentions flight, radar, turbulence, and
63. (C) The woman is shopping for seatbelts. Choice (A) is a different form of
envelopes. Choice (A) confuses book with transportation. Choice (B) associates
the similar-sounding word box. Choice airport with airplane. Choice (D) is a
(B) confuses adhesive tape with self- different form of transportation.
adhesive envelopes. Choice (D) confuses 72. (A) The speaker says This is your captain.
letter paper with business-letter size Choice (B) associates tour guide with what
envelopes. the passengers can see from the plane.
64. (A) The man offers to order the kind of Choice (C) associates weather forecaster
envelope that the woman wants. Choice with turbulence. Choice (D) associates
(B) is the kind of envelope that is in the flying with passenger.
store. Choice (C) is what the woman says 73. (D) The speaker says You can see Mt.
she will do. Choice (D) repeats the word Rushmore. Choice (A) confuses cemetery
Copyright © 2012 by Pearson Education, Inc. Permission granted to reproduce for classroom use.

box. with Deadwood. Choice (B) uses radar in a


65. (D) The man says they answered ten different context. Choice (C) confuses
phone calls. Choice (A) confuses two with woods with Deadwood.
to. Choice (B) confuses four with for. 74. (A) The speaker says Welcome aboard our
Choice (C) is not mentioned. Tropical Garden Tour. Choice (B) associates
66. (C) The woman says that she is expecting forest with large tree branches. Choice (C)
a package. Choices (A), (B), and (D) are associates museum with collection. Choice
things the man mentioned when (D) associates flower shop with flowers.
describing the work they had already 75. (C) The speaker asks participants to
done. remain seated. Choice (A) is what the tour
67. (A) The man asks the woman to help him guide asks participants not to do. Choice
write a report. Choice (B) associates mail (B) confuses close with the similar-
with letters. Choice (C) is confused with sounding clothes. Choice (D) confuses
the woman’s mention that the man’s clean the windows with keep from leaning
office is cleaner. Choice (D) repeats the out the windows.
word computer. 76. (B) The speaker says the first stop is on
68. (C) The woman pays with a $100 bill. your left. Choice (A) mentions trees in a
Choice (A) uses the word check in a different context. Choice (C) mentions
different context. Choice (B) is the way river branches not tree branches. Choice (D)
the woman originally wants to pay. confuses rear with the similar-sounding
rare.

INTRODUCTORY COURSE ANSWER KEY: LISTENING COMPREHENSION REVIEW 10

Downloaded by Minh Nguy?n V?n (nguyenvanminh07012003@gmail.com)


lOMoARcPSD|9770843

77. (D) The speaker mentions a community 86. (B) The speaker mentions heavy rains.
clean-up drive. Choice (A) associates Choices (A), (C), and (D) are all things
garbage collectors with trash and litter. that may happen.
Choice (B) repeats the word community. 87. (C) The speaker mentions problems
Choice (C) repeats the words kids and during the next 24 hours. Choices (A),
Saturdays. (B), and (D) are not mentioned.
78. (C) The speaker says the clean-up was a 88. (C) The speaker says to listen for
dismal failure and only 15 people showed up. evacuation orders. Choice (A) confuses
Choice (A) is not mentioned in relation to vacation orders with the similar-sounding
last year’s drive. Choice (B) confuses 50 evacuation orders. Choice (B) confuses
with the similar-sounding 15. Choice (D) train with the similar-sounding rain.
is not correct because the speaker Choice (D) is not mentioned.
mentions 15 people.
89. (B) The speaker mentions that the food
79. (A) The speaker says We’ll start at ten in was paid for thanks to a slight increase in
the morning. Choice (B) is when the drive revenue over the past year. Choice (A) is
will finish. Choice (C) confuses three with not mentioned. Choice (C) confuses
the similar-sounding free. Choice (D) generosity with thanks. Choice (D) is not
confuses three with the similar-sounding mentioned.
free and repeats 15 in a different context.
90. (A) The speaker mentions that the
80. (C) The speaker says that Dr. Jones is a purpose of the talk is to plan for the
professor of economics. Choice (A) is what future. Choice (B) is not mentioned.
the announcer is. Choice (B) associates Choice (C) is not the main purpose of the
Dr. with medicine. Choice (D) associates speech. Choice (D) is not mentioned.
farmer with agricultural sector.
91. (B) The speaker says in the afternoon
81. (B) The speaker says We will read from they will meet in groups. Choice (A)
your letters. Choices (A) and (C) are topics confuses write new ads with radio ads.
the guest will talk about. Choice (D) Choice (C) confuses visit customers’ homes
associates farmer with agricultural. with more door-to-door contact. Choice (D)
82. (C) The speaker mentions the next hour. is not mentioned.
Copyright © 2012 by Pearson Education, Inc. Permission granted to reproduce for classroom use.

Choice (A) is how long the announcer 92. (C) The speaker says people couldn’t stay
will read letters and e-mails. Choice (B) is dry and interiors became like ovens. Choices
how long the guest will be on the show. (A), (B), and (D) are not mentioned.
Choice (D) is not the correct total.
93. (A) The speaker says by the end of the year.
83. (C) The speaker says Good afternoon, class. Choice (B) is the beginning of another
Choice (A) will be next week’s special year. Choices (C) and (D) are not
guest. Choice (B) is who will come next mentioned.
week. Choice (D) confuses the speaker
with the class. 94. (D) The speaker mentions special
arrangements with a shopping center to
84. (A) The speaker mentions ancient Chinese use one of their lots. Choices (A), (B), and
history. Choice (B) confuses art history (C) are not mentioned.
with art. Choice (C) confuses writing with
the related word written. Choice (D) 95. (C) The number is 603-555-9000. Choices
confuses travel with the guest’s trip. (B), (C), and (D) are incorrect.

85. (B) The speaker says they will see slides 96. (D) The speaker says We’re real estate
instead of the video. Choice (A) is not investors. Choice (A) confuses investment
mentioned. Choice (C) is what they were bankers with investors. Choice (B)
going to do, but the video is unavailable. confuses real estate company with real
Choice (D) associates art museum with art. estate investors. Choice (C) is what the
speaker says they are not.

INTRODUCTORY COURSE ANSWER KEY: LISTENING COMPREHENSION REVIEW 11

Downloaded by Minh Nguy?n V?n (nguyenvanminh07012003@gmail.com)


lOMoARcPSD|9770843

97. (A) The speaker wants to buy houses


from people. Choices (B), (C), and (D) are
all people who might want to buy a
house, not sell a house.
98. (C) The speaker mentions frozen food, aisle
10, apples, pears, and fresh fruit, all things
in a grocery store. Choice (A) associates a
farm with apples, pears, and fresh fruit.
Choice (B) confuses theater with the
similar-sounding words there’s a. Choice
(D) associates cell phone store with cell
phone.
99. (D) The speaker mentions going to the
customer service office to claim it.
Choices (A), (B), and (C) are all
mentioned, but they are not the place to
claim the lost cell phone.
100. (A) The speaker says Only customers with
a Shoppers’ Club membership. Choices (B),
(C), and (D) are mentioned, but they are
not correct answers.
Copyright © 2012 by Pearson Education, Inc. Permission granted to reproduce for classroom use.

INTRODUCTORY COURSE ANSWER KEY: LISTENING COMPREHENSION REVIEW 12

Downloaded by Minh Nguy?n V?n (nguyenvanminh07012003@gmail.com)


lOMoARcPSD|9770843

ANSWER KEY
READING
PART 5: INCOMPLETE SENTENCES Verb Tenses (page 136)
Word Families (page 122) 1. (B) 6. (A)
2. (C) 7. (C)
1. (A) 6. (D)
3. (C) 8. (B)
2. (B) 7. (C)
4. (D) 9. (A)
3. (C) 8. (B)
5. (C) 10. (D)
4. (C) 9. (A)
5. (B) 10. (C) Phrasal Verbs (page 138)
Similar Words (page 125) 1. (B) 6. (C)
2. (C) 7. (B)
1. (C) 6. (D)
3. (D) 8. (A)
2. (D) 7. (B)
4. (D) 9. (C)
3. (C) 8. (A)
5. (B) 10. (A)
4. (B) 9. (A)
5. (D) 10. (D) Strategy Practice (page 141)
Pronouns (page 127) 1. (C) 11. (A)
2. (B) 12. (D)
1. (D) 6. (C)
3. (D) 13. (B)
2. (D) 7. (A)
4. (C) 14. (C)
3. (A) 8. (C)
5. (B) 15. (D)
4. (B) 9. (D)
6. (B) 16. (C)
5. (B) 10. (C)
7. (A) 17. (A)
Prepositions (page 129) 8. (D) 18. (B)
1. (B) 6. (C) 9. (B) 19. (C)
Copyright © 2012 by Pearson Education, Inc. Permission granted to reproduce for classroom use.

2. (D) 7. (C) 10. (C) 20. (A)


3. (C) 8. (A)
4. (A) 9. (B) PART 6: TEXT COMPLETION
5. (D) 10. (C) Words in Context (page 144)
Coordinate Conjunctions (page 131) 1. (D) 6. (A)
1. (D) 6. (B) 2. (A) 7. (B)
2. (A) 7. (A) 3. (B) 8. (B)
3. (A) 8. (C) 4. (B) 9. (D)
4. (B) 9. (B) 5. (B) 10. (C)
5. (A) 10. (A) Possessive Adjectives and Pronouns
Subordinate Conjunctions (page 134) (page 146)
1. (D) 6. (B) 1. (A) 6. (B)
2. (C) 7. (A) 2. (B) 7. (A)
3. (A) 8. (C) 3. (C) 8. (B)
4. (B) 9. (B) 4. (C) 9. (D)
5. (A) 10. (A) 5. (D) 10. (A)

INTRODUCTORY COURSE ANSWER KEY: READING 13

Downloaded by Minh Nguy?n V?n (nguyenvanminh07012003@gmail.com)


lOMoARcPSD|9770843

Prepositions: Time (page 148) 25. (C) 43. (A)


26. (D) 44. (D)
1. (B) 6. (B)
27. (D) 45. (B)
2. (B) 7. (A)
28. (D) 46. (C)
3. (C) 8. (D)
29. (C) 47. (B)
4. (A) 9. (C)
30. (B) 48. (C)
5. (D) 10. (B)
31. (A) 49. (A)
Verb Tense: Time Clauses (page 150) 32. (D) 50. (D)
1. (B) 6. (C) 33. (D) 51. (A)
2. (D) 7. (D) 34. (A) 52. (D)
3. (A) 8. (B) 35. (B) 53. (B)
4. (C) 9. (A) 36. (D) 54. (C)
5. (A) 10. (C) 37. (D) 55. (D)
38. (B) 56. (B)
Adjective Comparisons (page 153) 39. (B) 57. (D)
1. (C) 6. (C) 40. (B) 58. (A)
2. (B) 7. (B) 41. (C) 59. (D)
3. (A) 8. (D) 42. (C) 60. (B)
4. (D) 9. (A)
Strategy Practice
5. (D) 10. (C)
Single Passages (page 189)
Gerunds or Infinitives (page 155) 1. (D) 11. (A)
1. (B) 6. (A) 2. (B) 12. (C)
2. (A) 7. (B) 3. (A) 13. (C)
3. (D) 8. (A) 4. (C) 14. (A)
4. (B) 9. (D) 5. (C) 15. (D)
5. (A) 10. (C) 6. (A) 16. (B)
Strategy Practice (page 158) 7. (B) 17. (D)
8. (A) 18. (C)
1. (A) 7. (D) 9. (D) 19. (D)
2. (D) 8 (A) 10. (B) 20. (A)
Copyright © 2012 by Pearson Education, Inc. Permission granted to reproduce for classroom use.

3. (A) 9. (C)
4. (A) 10. (B) Double Passages (page 196)
5. (D) 11. (C) 21. (C) 26. (A)
6. (B) 12. (B) 22. (C) 27. (A)
23. (A) 28. (C)
PART 7: READING COMPREHENSION 24. (D) 29. (D)
(PAGE 167) 25. (A) 30. (C)
1. (B) 13. (B)
2. (D) 14. (D)
3. (A) 15. (B)
4. (D) 16. (D)
5. (C) 17. (B)
6. (C) 18. (C)
7. (D) 19. (D)
8. (A) 20. (B)
9. (A) 21. (B)
10. (C) 22. (B)
11. (C) 23. (C)
12. (B) 24. (B)

INTRODUCTORY COURSE ANSWER KEY: READING 14

Downloaded by Minh Nguy?n V?n (nguyenvanminh07012003@gmail.com)


lOMoARcPSD|9770843

ANSWER KEY
READING REVIEW
PART 5 (PAGE 200) 108. (C) At is the preposition used when
specifying an exact time. Choices (A) and
101. (D) A past participle is required after were
(B) are prepositions that are not used
to make the passive voice: were organized.
with time phrases containing hours and
Choices (A) and (B) are nouns. Choice (C)
minutes. Choice (D) is used with nouns
is the base verb form.
that represent a period of time: the winter,
102. (C) A past participle adjective describes the war, the meeting.
the noun sales plan. Choice (A) is the base
109. (A) Instructions, which means “how to do
verb form. Choice (B) is the simple
something,” is the only noun that is
present. Choice (D) is the present
logical for this sentence. Choice (B),
participle.
delays, means “postponements.” Choice
103. (B) The subordinate conjunction because (C), reservations, means “something to be
is needed to explain why the action was set aside for future use.” Choice (D),
taken. Choice (A) goes with the other adjustments, means “changes made to
clause: There were many customer correct something.”
complaints; therefore, we withdrew the item
110. (C) A past participle is required here. It
from the market. Choice (C) is contrary to
represents a reduced passive voice: any
the idea. Choice (D) doesn’t make sense
other project that has been undertaken.
in this context.
Choice (A) is a present participle. Choice
104. (A) A past participle is required after is to (B) is the simple past. Choice (D) is the
make the passive voice: is respected. base verb form.
Choice (B) is the simple present. Choice
111. (A) An adverb is required to describe
(C) is the base verb form. Choice (D) is
the adjective changing. Choice (B) is a
the present participle.
Copyright © 2012 by Pearson Education, Inc. Permission granted to reproduce for classroom use.

gerund. Choice (C) is the simple present.


105. (C) Increased, which means “made more Choice (D) is the simple past.
or longer” in this case, is the only idea
112. (C) The preposition beyond is the only one
that is logical for this sentence. Choice
that is logical in this sentence. In this case
(A), insisted, means “demanded” or
it means “more/greater than.” Choice (A),
“required.” Choice (B), installed, means
outside, means “not related to” in this
“put in place.” Choice (D), intruded,
context. Choice (B), inside, means “agrees
means “put or forced in without being
with” in this context. Choice (D), before, is
asked or wanted.”
not logical in this idea.
106. (B) A noun used as an adjective, sales, is
113. (A) An adjective is required to describe
required to explain what the report is
the noun demands. Choice (B) is an
about. Choice (A) is a present participle.
adverb. Choice (C) is a noun. Choice (D)
Choice (C) is the base verb form. Choice
is a gerund.
(D) is the simple present.
114. (D) The noun which means “the people
107. (B) A singular noun is required because
who plan something ” is required as the
of the article a. Choice (A) is a gerund but
subject in this sentence. Choice (A) is the
has a different meaning. Choice (C) is a
noun for things. Choice (B) is the present
plural noun. Choice (D) is the simple
participle. Choice (C) is the simple past.
past.

INTRODUCTORY COURSE ANSWER KEY: READING REVIEW 15

Downloaded by Minh Nguy?n V?n (nguyenvanminh07012003@gmail.com)


lOMoARcPSD|9770843

115. (B) The third person singular, masculine 123. (B) A noun is required after the definite
possessive adjective is required. Choice article the. In this case we need the noun
(A) is the direct object form. Choice (C) is that means “the event.” Choice (A) is the
the subject form. Choice (D) is the simple past. Choice (C) is the noun for a
reflexive pronoun. person. Choice (D) is the simple present.
116. (A) Indications, which means “things that 124. (A) The expression to have a clue means
show you something,” is the only logical “to understand” or “to have an idea
idea for this sentence. Choice (B), about something.” Choices (B), (C), and
solutions, means “ways to solve a (D) carry the basic idea needed but are
problem.” Choice (C), proposals, means not used in this expression.
“suggestions.” Choice (D), revisions, 125. (C) The preposition without is required
means “changes to improve something.” with the noun phrase a good credit history.
117. (D) Enter, which means “putting Choices (A) and (D) are illogical in this
information into a computer,” is the only idea. Choice (B) is a conjunction.
logical idea for this sentence. Choice (A), 126. (C) A verb is required after the subject of
submerge, means “to put or go under the sentence. Choices (A) and (D) are
water.” Choice (B), propose, means “to nouns. Choice (B) is the present
suggest.” Choice (C), admit, means “to participle.
allow to enter or pass.”
127. (A) The reflexive pronoun is used here to
118. (C) The subordinate conjunction although show that the subject does the action
is required to introduce this clause. It alone. Choice (B) is a subject pronoun.
means that what follows is contrary to a Choice (C) is an object pronoun. Choice
logical conclusion. Choices (A) and (D), (D) is a possessive pronoun.
which carry the idea of but, would work
at the beginning of the second clause. 128. (D) Minimize, which means “to make as
Choice (B) is illogical in this context. little or small as possible,” is the only
verb that is logical in this sentence.
119. (A) When negative adverbs of frequency Choice (A), criticize, means “to judge the
are used at the beginning of a sentence, good or bad points of something.”
there is a word inversion that looks like a Choice (B), localize, means “to restrict to a
Copyright © 2012 by Pearson Education, Inc. Permission granted to reproduce for classroom use.

question form: Never have the employees particular area.” Choice (C), sanitize,
complained . . . Choices (B), (C), and (D) means “to make clean and hygienic.”
are not negatives and are also illogical for
this idea. 129. (C) Impatient, which means “not willing
to wait,” is the only adjective that is
120. (D) The preposition from explains where logical in this sentence. Choice (A),
these people are originating. Choices (A), evident, means “obvious, clear.” Choice
(B), and (C) are not logical prepositions (B), extraordinary, means “more than
for describing these people. usual, special.” Choice (D), inconclusive,
121. (B) An adverb is required to describe means “not completely sure or proven.”
how the crew worked. Choice (A) is an 130. (C) A noun is required before the verb
adjective. Choice (C) is a noun or verb. broke down. Choice (A) is an adjective.
Choice (D) is a comparative adjective Choice (B) is the simple past. Choice (D)
phrase. is the base verb form.
122. (C) An adjective is required to describe
the noun phrase job performance. Choice
(A) is a preposition. Choice (B) is a noun.
Choice (D) is an adverb.

INTRODUCTORY COURSE ANSWER KEY: READING REVIEW 16

Downloaded by Minh Nguy?n V?n (nguyenvanminh07012003@gmail.com)


lOMoARcPSD|9770843

131. (D) The pronoun what, which means “the sentence. Choice (A), alteration, means “a
particular thing that . . . ,” is required as change, making something different.”
the subject. Choice (A) is a pronoun but Choice (C), anxiety, means “a state of
means something already indicated, so it uneasiness and worry.” Choice (D),
doesn’t work here. Choice (B) is a ambivalence, means “feeling two different
pronoun but means a particular thing in ways about something or somebody at the
a selection, so it doesn’t work here. same time.”
Choice (C) is a pronoun referring to 138. (C) The construction so . . . that . . .
people, so it doesn’t work here. quantifies how something was. So is
132. (A) Proposed, which means “suggested” always followed by an adjective or
or “asked to be considered,” is the only adverb. Choice (A) has the same
one of these past participles that is construction, such . . . that . . . , but such is
logical for the idea of this sentence. followed by a noun phrase: such a hot day;
Choice (B), propelled, means “caused to such a well-written report. Choices (B) and
be set in motion.” Choice (C), preferred, (D) are not logical for the idea of this
means “liked the most.” Choice (D), sentence.
preordained, means “made to be 139. (D) A noun for a thing must be used as
someone’s fate or destiny.” the subject of this sentence. Choice (A) is
133. (B) The subject of this sentence must be a a noun but means the person. Choice (B)
noun. Choice (A) is an adjective. Choice is the base verb form. Choice (C) is an
(C) is both the simple past and the past adjective.
participle adjective. Choice (D) is the 140. (C) Therefore is a conjunction meaning
present participle. “because of what I have just said” or “for
134. (A) Refrain, which means “to keep or stop this reason.” Choices (A), (B), and (D) are
from doing something,” is the only conjunctions but are not logical for the
logical idea for this sentence. Choice (B), idea of this sentence.
respect, means “to have high regard or
esteem for.” Choice (C), reserve, means PART 6 (PAGE 204)
“to keep for a special use.” Choice (D), 141. (C) We know that the writer is currently
restore, means “to bring back to the at his new job because of the use of the
Copyright © 2012 by Pearson Education, Inc. Permission granted to reproduce for classroom use.

original condition.” present perfect tense in the first sentence:


135. (C) The present participle, or –ing, My first week . . . has been very good.
adjective is required to describe the news. Therefore, the present progressive form
Use this adjective form when the person am enjoying is correct. Choice (A) is
or thing described is creating the feeling. simple past. Choice (B) is future. Choice
Choice (A) is the base verb form. Choice (D) is past progressive.
(B) is the simple past or past participle 142. (A) First introduces the first problem that
adjective. Choice (D) is the simple the writer mentions. Choices (B), (C), and
present. (D) are words to introduce items that
136. (D) Inadequate, which means follow the first item on a list.
“insufficient” or “not enough,” is the 143. (B) The adverb slowly describes how the
only adjective that is logical to complete computer works. Choice (A) is an
the idea of this sentence. Choice (A), adjective. Choice (C) is a comparative
improper, means “not correct or adjective. Choice (D) is a noun.
appropriate.” Choice (B), unlikely, means
“improbable.” Choice (C), unlucky, means 144. (B) This announcement is about helping
“not having luck.” people who have suffered during a
disaster, in other words, victims. Choice
137. (B) Attitude, which means “a state of (A) means “people who watch
mind” or “feeling,” is the only noun that something.” Choice (C) means “people
is logical to complete the idea of this

INTRODUCTORY COURSE ANSWER KEY: READING REVIEW 17

Downloaded by Minh Nguy?n V?n (nguyenvanminh07012003@gmail.com)


lOMoARcPSD|9770843

who plan something.” Choice (D) means PART 7 (PAGE 207)


“people who tell about what happened.”
153. (B) Professionals is the correct answer
145. (D) A noun is needed here as an object of because all the specific information given
the verb lost. Choice (A) is a present tense is about professionals. Choice (A)
verb. Choice (B) is a past tense verb. associates laborers with work. Choice (C)
Choice (C) is a present participle. is incorrect because the information is
146. (D) The announcement asks people to specifically about professional workers.
give money, or a donation, to help the Choice (D) is the people who did the
flood victims. Choices (A), (B), and (C) study.
are things one could make, but they do 154. (A) The report states that Ten years ago,
not fit the context. most professional people worked 40 hours a
147. (D) Mr. Miser’s prices are compared to all week or less, so the correct answer is 40 or
the other prices in town, so the less. Choices (B), (C), and (D) confuse
superlative form of the adjective is other numbers in the report.
required here. Choice (A) is a simple 155. (B) The numbers were released last week,
adjective form. Choice (B) is a so the correct answer is 7 days ago. Choice
comparative adjective. Choice (C) looks (A) is not mentioned. Choices (C) and (D)
like a comparative adjective but is confuse other times mentioned in the
actually a present tense verb. report.
148. (C) This part of the ad talks about the 156. (C) 100 is the correct answer. Choices (A),
places, or locations, where you can find (B), and (D) are the number of liters sold
Mr. Miser’s agencies. Choices (A), (B), in other months.
and (D) mention other aspects of Mr.
157. (C) May is the correct answer. Choices
Miser’s car rental business.
(A), (B), and (D) are amounts of ice cream
149. (A) This is an imperative sentence, which during other months.
uses the base form of the verb. Choice (B)
158. (B) February is the correct answer. Ike
is the simple tense. Choice (C) is the
sold only 90 liters of ice cream during
present participle. Choice (D) is the
that month, but he sold larger amounts
future.
during all the other months. Choices (A),
Copyright © 2012 by Pearson Education, Inc. Permission granted to reproduce for classroom use.

150. (A) Turn off means to stop the electricity. (C), and (D) are other months on the
Choices (B), (C), and (D) would form graph.
phrasal verbs with meanings that do not
159. (C) The graph covers the six months from
fit the context of the sentence.
January through June, so the correct
151. (B) Inconvenience means problems or answer is half a year. Choices (A), (B), and
difficulties. Choices (A), (C), and (D) look (D) are incorrect.
similar to the correct answer but have
160. (A) In the first sentence of the letter,
meanings that don’t fit the context.
Charles Chung states I am interested in
152. (A) Patience is a noun following the working at the Worldwide Travel Agency, so
preposition for. Choice (B) is also a noun He wants a job is the correct answer.
but has a different meaning. Choice (C) is Choice (B) confuses experience working
an adverb. Choice (D) is an adjective, or with computers with wanting to buy a
the singular form of the noun patients. computer. Choice (C) associates the
travel agency with taking a trip. Choice

INTRODUCTORY COURSE ANSWER KEY: READING REVIEW 18

Downloaded by Minh Nguy?n V?n (nguyenvanminh07012003@gmail.com)


lOMoARcPSD|9770843

(D) confuses enclosing my résumé with (B) associates the name of the market
writing a résumé. ABC with school. Choice (C) associates
food with restaurant. Choice (D)
161. (D) The letter states I have five years’
associates cookies with bakery.
experience as a travel agent, so the correct
answer is He worked as a travel agent. 170. (C) The sale is this weekend only, so the
Choice (A) is incorrect because we don’t correct answer is Saturday and Sunday.
know how long he studied computers. Choices (A), (B), and (D) confuse the
Choice (B) is incorrect because he wants days the store is open.
to work with Ms. Greene in the future, 171. (C) Fresh produce is produce that has just
but he hasn’t worked with her yet. been picked. The other options do not fit
Choice (C) associates the travel agency the context.
with traveling.
172. (B) $2.50 a bag is the correct answer.
162. (C) The announcement says Our new Choices (A) and (C) are not mentioned.
address, starting April 12, so the correct Choice (D) is incorrect because a
answer is April 12. Choices (A), (B), and customer has to pay for two bags before
(D) confuse 12 with similar-looking getting one bag for free.
numbers.
173. (D) 45% chose soccer as their favorite
163. (A) On Oakland Avenue is the correct sport. Choices (A), (B), and (C) have
answer. Choice (B) confuses the street lower percentages.
address with the suite number. Choice
(C) is a nearby street. Choice (D) confuses 174. (B) 20% prefer basketball. Choices (A),
across the street from a bank with next to a (C), and (D) match other sports on the
bank. chart.

164. (C) The announcement says Our phone 175. (C) This is a form for subscribing to
number will stay the same, so the correct something that will arrive in your home
answer is the telephone number. Choices monthly, so the correct answer is a
(A), (B), and (D) are all things that will magazine. Choices (A), (B), and (D) are
change. things that one does not normally
subscribe to.
165. (B) Three hours is the correct answer.
Copyright © 2012 by Pearson Education, Inc. Permission granted to reproduce for classroom use.

Choices (A) and (C) confuse other 176. (B) Helena Bishop is ordering a one-year
periods of time mentioned in the article. subscription. The cost for one year is $45.
Choice (D) is the length of the trip on the Choice (A) is the cost for six months.
regular train. Choice (C) is the cost for two years.
Choice (D) is not mentioned.
166. (C) Four is the correct answer. Choices
(A), (B), and (D) confuse other numbers 177. (C) Helena marked credit card on the
mentioned in the article. form, so the correct answer is credit card.
Choices (A), (B), and (D) are methods she
167. (B) The article states Many people will did not choose.
prefer the regular trains because the tickets
are cheaper, so The tickets are less expensive 178. (D) Employees who wish to smoke must
is the correct answer. Choices (A), (C), go outdoors but not near the main
and (D) mention things that are true entrance. Choice (A) associates picnic area
about the express trains, not the regular with outdoors. Choices (B) and (C) are not
trains. mentioned.

168. (C) The train will start operating between 179. (B) Employees who smoke feel like they
the two cities. The other options do not are being treated like second-class
fit the context. citizens. Choice (A) is not mentioned.
Choice (C) is incorrect because there is no
169. (A) The business sells fruit and meat by employee lounge; it is something that’s
the kilo and cookies by the bag, which being suggested. Choice (D) confuses the
are things sold at a grocery store. Choice

INTRODUCTORY COURSE ANSWER KEY: READING REVIEW 19

Downloaded by Minh Nguy?n V?n (nguyenvanminh07012003@gmail.com)


lOMoARcPSD|9770843

weather making it hard to work and having to 188. (A) Elizabeth’s class begins at 6:00, and
smoke outdoors in the bad weather. she says that that gives her half an hour
to get there after leaving work, so she
180. (B) Ms. Dunnaway is requesting a
leaves work at 5:30. Choices (B), (C), and
meeting with Ms. Fagan for the
(D) are not correct.
employees who smoke. Choice (A)
confuses the request and Ms. Fagan’s 189. (D) Elizabeth says that she wants to take
decision to make the mill off limits to a word processing class that begins at
smokers. Choice (C) is incorrect because 7:30 on Tuesdays and Thursdays. By
she suggests one employee lounge in her looking at the schedule we see that the
letter. Choice (D) is incorrect because the only word processing class at that time is
location of the meeting is not mentioned. Level IV. Choices (A), (B), and (C) are not
correct.
181. (D) The e-mail reports that the location of
the meeting was changed to the lounge. 190. (D) Elizabeth mentions two classes that
Choice (A) was the original location of she wants to take. By looking at the
the meeting. Choice (B) is not mentioned. schedule, we see that each class costs
Choice (C) is confused with the agenda $275, making the total cost for two
item about conference plans. classes $550. Choices (A), (B), and (C) are
not correct.
182. (C) The e-mail says that the third item on
the agenda was discussed first. By 191. (A) Although the memo says that both
looking at the agenda, we see the new conference rooms need painting, it only
hiring policy is the third item. Choices (A), mentions one that will actually be
(B), and (D) are the other agenda items. painted now. Choice (B) is the number of
conference rooms that need painting.
183. (A) The e-mail, which is sent to Sam Blair,
Choices (C) and (D) are other rooms
states that everyone was there, except you.
mentioned in the memo and e-mail.
Choice (B) wrote the e-mail. Choices (C)
and (D) are names mentioned on the 192. (A) The memo states that the painting
meeting agenda who attended the should take no more than two days. Choices
meeting. (B), (C), and (D) are not correct.
184. (B) According to the e-mail, the meeting 193. (D) In the e-mail, George explains why he
Copyright © 2012 by Pearson Education, Inc. Permission granted to reproduce for classroom use.

lasted two and a half hours. Choice (A) needs to use the conference room and
looks similar to the correct answer. then asks the question Would it be possible
Choices (C) and (D) are confused with to schedule the painting so that it begins on
the time the meeting started. Wednesday or Thursday? Choice (A) is
confused with George wanting to use the
185. (D) The e-mail says that next month’s
conference room for a meeting. Choice
meeting will be held on the 18th. We see
(B) is confused with George’s mention of
on the agenda that this month is April, so
his office. Choice (C) is confused with
the next meeting will take place on May
George’s explanation of why he can’t use
18th. Choices (A), (B), and (C) are not
other rooms.
correct.
194. (A) In his e-mail George says that his
186. (D) The schedule lists Monday–
meeting is planned for the day painting
Wednesday classes, Tuesday–Thursday
begins. We know from the first memo that
classes, and one Saturday class. That
the painting will begin on Tuesday.
adds up to five days. Choices (A), (B),
Choices (B) and (C) are the days George
and (C) are not correct.
suggests that the painting begin. Choice
187. (B) The instructions following the class (D) is confused with Conference Room 2 is
schedule say to mail a completed already booked through Friday.
registration form to the school. Choices
(A), (C), and (D) are not mentioned.

INTRODUCTORY COURSE ANSWER KEY: READING REVIEW 20

Downloaded by Minh Nguy?n V?n (nguyenvanminh07012003@gmail.com)


lOMoARcPSD|9770843

195. (C) In his e-mail George says the meeting 198. (C) The letter states you have the
place you suggest is too informal. By educational level we require. By looking at
looking at the first memo, we see that the ad, we see that the required
Luis suggests using the cafeteria for educational level is a Master’s in
meetings while the conference room is Business Administration. Choice (A) is
being painted. Choices (A), (B), and (D) confused with Ms. Choi’s Bachelor’s
are places mentioned in the memo and degree in Psychology. Choice (B) is
e-mail, but they are not the correct confused with the business degree and
answers. the title of the position associate
researcher. Choice (D) is confused with
196. (B) The ad gives Ms. Kovacs’ title as
the Master’s degree in Business
Director of Human Resources. Choice (A) is
Administration.
the title of the advertised position.
Choice (C) is confused with the team of 199. (D) The letter states that Ms. Choi has
market researchers. Choice (D) is confused more years of experience in the field than we
with Ms. Choi’s psychology degree. asked for, and we see in the ad that three
to five years are required. Choices (A),
197. (D) The letter states that the position that
(B), and (C) are confused with the actual
we advertised has already been filled. Choice
number of years required.
(A) is not correct because we can tell that
Ms. Kovacs has read the résumé by her 200. (B) At the end of the letter Ms. Kovacs
knowledge of Ms. Choi’s background. suggests that Ms. Choi call her in six
There is nothing to indicate that choice months. Choices (A), (C), and (D) are
(B) is true. Choice (C) is not true because confused with other things mentioned in
Ms. Kovacs points out several the letter.
qualifications in the letter.
Copyright © 2012 by Pearson Education, Inc. Permission granted to reproduce for classroom use.

INTRODUCTORY COURSE ANSWER KEY: READING REVIEW 21

Downloaded by Minh Nguy?n V?n (nguyenvanminh07012003@gmail.com)


lOMoARcPSD|9770843

ANSWER KEY
PRACTICE TEST ONE
PART 1 (PAGE 226) 6. (D) The man is drinking from a teacup.
Choice (A) provides incorrect details: He
1. (B) The cars are on the road. Choice (A)
is holding a cup, not holding his head.
tries to confuse you by using the similar-
Choice (B) provides incorrect details: He
sounding word carts for cars. Choice (C)
is wearing glasses, not covering his glasses.
provides incorrect details: The drivers
Choice (C) provides incorrect details:
are inside driving the cars, they’re not by
There are napkins on the table, but he is
the car. Choice (D) tries to confuse you by
not using them.
using the similar-sounding cards for cars.
7. (B) The three men are talking to a female
2. (A) We can assume the men in uniform
colleague. Choice (A) provides incorrect
are the ship’s officers and they are on the
details: The office is small and cramped.
boat. Choice (B) tries to confuse you by
Choice (C) provides incorrect details: The
using the similar-sounding word sheep
men are looking at the woman, not a
for ship. Choice (C) tries to confuse you
computer game. Choice (D) tries to confuse
by using the similar-sounding word
you by using the similar-sounding word
sheets for ship and by confusing sale for
headquarters for heads.
sail. Choice (D) mentions sailors, but they
are not playing bridge. 8. (A) The man is working on a construction
site and wearing a hard hat. Choice (B) is
3. (A) The woman is talking on the phone.
the opposite of what the photo shows:
Choice (B) provides incorrect details: An
the man is outside, not inside. Choice (C)
open book is on the table, but there is no
is also the opposite of what the photo
open drawer. Choice (C) provides incorrect
shows: the man is looking up, not down at
details: She has a pen in her hand and an
the ground. Choice (D) mentions the
open book in front of her, but she is not
man’s gloves, but he is wearing them, not
Copyright © 2012 by Pearson Education, Inc. Permission granted to reproduce for classroom use.

writing in it. Choice (D) provides


buying them.
incorrect details: There is a keyboard in
the background, but she is not using it. 9. (D) The man is checking into a hotel.
Choice (A) tries to confuse you by using
4. (B) Several chefs are in a kitchen
the related words checking and counter.
preparing food. Choice (A) correctly
Incorrect details are provided: There are
identifies the vegetable, but incorrectly
no bags in the picture. Choice (B)
identifies the occupation and activity of
provides incorrect details: The man is not
the people. Choice (C) incorrectly
ordering room service. Choice (C) tries to
identifies the occupation of the people.
confuse you by using related words
Choice (D) associates waiters and meal
elevator and room.
with the scene, but the meal is not ready
to be served. 10. (B) The woman is faxing a document.
Choice (A) provides incorrect details: the
5. (D) The men are digging a hole in the
woman’s hair covers her face, but the
ground. Choice (A) provides incorrect
woman is not covering her face. Choice (C)
details: They’re standing, not sitting.
confuses by using the related word photo,
Choice (B) provides incorrect details:
which refers to a different piece of office
They’re digging near the fence, not climbing
equipment, a photocopy machine. Choice
the fence. Choice (C) incorrectly associates
(D) provides incorrect details: She’s
watering plants with digging in the garden.
putting the document in the document feeder,
not holding open a door.

INTRODUCTORY COURSE ANSWER KEY: PRACTICE TEST ONE 22

Downloaded by Minh Nguy?n V?n (nguyenvanminh07012003@gmail.com)


lOMoARcPSD|9770843

PART 2 (PAGE 232) dripping faucet. Choice (A) repeats the


word drive but not in the idiomatic sense
11. (B) At 12:30 answers when. Choice (A)
of being driven crazy. Choice (C) uses the
associates airport with fight and answers
similar-sounding word tripped for
where. Choice (C) associates Gate with
dripping.
flight but answers where.
22. (A) There was a lot of traffic is a reason for
12. (A) I’ve got it [the fax] right here answers
being late. Choice (B) answers will you be
have we received the fax? Choice (B) In
late. Choice (C) confuses similar sounds
about an hour answers when. Choice (C)
ate and late and the opposites quickly
repeats the word fax but answers when
and late.
will you fax it?
23. (B) Yes, I sent the invitations yesterday
13. (C) How do you do answers the question.
answers the yes/no question and provides
Choice (A) confuses similar sounds
the day they were sent. Choice (A) is a
mistake and Ms. Pak. Choice (B) answers
possible response to an invitation. Choice
what do you do.
(C) repeats the word remember and
14. (C) Yes, [it is time for our meeting] but it’s every[body] but does not answer the
been rescheduled answers the question. question.
Choice (A) confuses similar sounds meat
24. (A) Yes, they’ve all gone out answers the
and meeting. Choice (B) answers when is
yes/no question. Choice (B) associates
our meeting.
sales with ads. Choice (C) repeats the
15. (B) I’ve seen him smiling answers the word newspaper and answers did you pick
yes/no question by associating smiling up the newspaper.
with being in a good mood. Choice (A)
25. (B) They’re going now answers when will
answers has the boss left. Choice (C)
the auditors be leaving. Choice (A) answers
answers where.
when did they leave. Choice (C) answers
16. (C) You should get more sleep responds to how long have they been here.
the statement I’m extremely tired. Choice
26. (C) is an affirmative response to the
(A) repeats the word extreme. Choice (B)
suggestion. Choice (A) confuses break
uses the similar-sounding word fire for
(chip or crack) with break (rest from work
tired.
Copyright © 2012 by Pearson Education, Inc. Permission granted to reproduce for classroom use.

or activity). Choice (B) confuses break


17. (C) That’s what I’ve been told answers the (not working) with break (rest from work
yes/no question by confirming the man or activity).
heard the Tokyo office would be opened.
27. (A) There were a lot of errors in it responds
Choice (A) uses the word open but
to the opinion about the article being
answers with the hours of operation.
poorly written. Choice (B) uses the word
Choice (B) repeats the word office but
rich for its opposite poor. However, in this
answers where are you going.
context poorly means not well. This choice
18. (B) We’re still waiting answers why. also uses the similar-sounding word
Choice (A) associates meeting with riding for written. Choice (C) uses the
conference but answers where. Choice (C) similar-sounding word art for article and
answers when. repeats the word poor.
19. (B) The new secretary answers Who are you. 28. (A) Canada answers Where do you come
Choice (A) answers how are you. Choice from? Choice (B) answers where did you
(C) answers where are you. come from specifically within the
20. (A) Ms. Hoffman’s answers whose building. Choice (C) answers when.
computer. Choice (B) answers which. 29. (B) I’d like to answers the invitation.
Choice (C) answers can I use it. Choice (A) The employee cafeteria’s
21. (B) So call a plumber identifies the downstairs answers where should we eat.
occupation of the person who can fix a Choice (C) associates the meal breakfast
with lunch.

INTRODUCTORY COURSE ANSWER KEY: PRACTICE TEST ONE 23

Downloaded by Minh Nguy?n V?n (nguyenvanminh07012003@gmail.com)


lOMoARcPSD|9770843

30. (B) You have the map upside down is a 40. (A) Yes, if you give us your receipts answers
possible reason that the street plan is will I be reimbursed. Choice (B) repeats the
confusing. Choice (A) repeats a form of words business trips and confuses similar
the verb find, found. Choice (C) repeats sounds expensive with expenses. Choice
the word plan and the associated phrase (C) associates afford with expenses but is
to look for for something you want to find. not a correct response.
31. (B) Not at all is a polite response to the
PART 3 (PAGE 233)
question. Choices (A) and (C) are not
appropriate responses to the question. 41. (C) The man says back here in the office.
Choice (A) associates mail with post office.
32. (A) About 12 hours answers how long.
Choice (B) confuses a park with Mr. Park.
Choice (B) answers how far. Choice (C)
Choice (D) is where the woman is going.
associates Egypt with Cairo and answers
where. 42. (D) The man says he mailed the contracts
this morning. Choice (A) is when the
33. (A) The boss asked me to answers Who are
woman will return. Choice (B) repeats the
you sending that e-mail for. Choice (C)
word morning. Choice (C) is not
answers who asked you to send the e-mail.
mentioned.
Choice (B) answers who are you sending
the e-mail to. 43. (C) The woman says it’s 11 o’clock already.
Choices (A) and (B) confuse 11 with the
34. (B) My new client answers who. Choice (A)
similar-sounding number 7. Choice (D)
answers where are you sending the e-mail
repeats the number 11.
from. Choice (C) associates stamps with
e-mail. 44. (D) The man wants to take the advanced-
level class. Choices (A), (B), and (C)
35. (B) Yes, that’s what the boss said answers
repeat other words mentioned in the
the yes/no question. Choice (A) answers
conversation but are not correct answers.
will stocks go up next year. Choice (C)
repeats the phrase next year. 45. (C) The woman says the tuition is $500.
Choice (A) confuses 90 with the similar-
36. (C) You’re right. I work too hard responds
sounding word fine. Choice (B) confuses
to the opinion You should take a vacation.
$400 with the similar-sounding word for.
Choice (A) repeats the words I take and
Copyright © 2012 by Pearson Education, Inc. Permission granted to reproduce for classroom use.

Choice (D) confuses $600 with six-month.


tries to confuse you with the similar-
sounding word transportation for vacation. 46. (A) The man will register for a morning
Choice (B) uses a form of the verb take class. Choices (B) and (C) confuses you
and the similar-sounding word station for with the similar-sounding word noon.
vacation. Choice (D) is not mentioned.
37. (C) Working overtime shouldn’t be a 47. (C) The man says He’ll be here tomorrow
problem. Choice (A) answers when the man morning. Choice (A) is when the woman
can start work but not if he can stay and thought Mr. Katz would arrive. Choices
work overtime. Choice (B) repeats the (B) and (D) are not mentioned.
word put but is in a different context. 48. (A) The woman heard about that terrible
38. (C) About 30 kilometers answers how much rainstorm. Choices (B) and (C) are other
farther. Choice (A) answers how much types of weather mentioned by the
longer. Choice (B) answers how much does speakers. Choice (D) confuses nice with
it cost. the similar-sounding word ice.
39. (B) I have to think about it some more is an 49. (B) The woman says she heard about the
appropriate response to the question. storm on the radio news. Choice (A)
Choice (A) confuses vacation with transfer. repeats a name mentioned elsewhere in
Choice (C) is a yes/no response but does the conversation. Choice (C) confuses
not fit the context. newspaper with news. Choice (D) is not
mentioned.

INTRODUCTORY COURSE ANSWER KEY: PRACTICE TEST ONE 24

Downloaded by Minh Nguy?n V?n (nguyenvanminh07012003@gmail.com)


lOMoARcPSD|9770843

50. (D) is the correct answer. Choice (A) is 59. (D) Mr. Cho retired. Choice (A) confuses
the place the speakers consider but reject. tired with the similar-sounding word
Choice (B) repeats the name of the party retired. Choice (B) repeats the word
honoree, but the party will not be held in vacation, which refers to Ms. Green, not
his office. Choice (C) repeats the word Mr. Cho. Choice (C) confuses fired with
party. the similar-sounding word retired.
51. (C) is the correct number of guests. 60. (D) Last month is when Mr. Cho retired.
Choices (A), (B), and (D) confuse the Choices (A) and (B) confuse two days and
correct answer with other similar- Tuesday with the similar-sounding word
sounding numbers. today. Choice (C) repeats the word week.
52. (D) The woman says the party will be on 61. (A) The woman says that without Mr.
Friday. Choice (A) confuses Tuesday with Cho, there’s no one here to tell us jokes.
two days away. Choice (B) confuses Choice (B) confuses worked hard with It’s
Wednesday with What day? Choice (C) is hard to work. Choice (C) confuses clean
not mentioned. with the similar-sounding word Green.
Choice (D) is incorrect because it is Ms.
53. (A) The speakers are talking about a table,
Green, not Mr. Cho, who brings cookies.
the woman says they are starving, and the
man mentions good food, so they are in a 62. (A) The man says they have been waiting
restaurant. Choices (B), (C), and (D) are for half an hour. Choice (B) repeats the
places where people might have to wait word hour. Choice (C) confuses 8 with the
and might need reservations. similar-sounding word late. Choice (D)
confuses 9 with the similar-sounding
54. (D) is the amount of time mentioned by
word time.
the man. Choices (A) and (C) confuse the
correct answer with other similar- 63. (C) The man tells us that Janet planned to
sounding numbers. Choice (B) confuses stop at the store. Choice (A) was Janet’s
ten with the similar-sounding word then. reason for being late last time. Choice (B)
repeats the words work and late in a
55. (C) The man says You could try that place
different context. Choice (D) confuses
around the corner. Choices (A) and (B)
exercise with the similar-sounding word
repeat other words mentioned in the
Copyright © 2012 by Pearson Education, Inc. Permission granted to reproduce for classroom use.

excuse.
conversation. Choice (D) is not
mentioned. 64. (B) The man says It’s really annoying.
Choice (A) confuses happy with the
56. (B) The man says that Mr. Curtis is out of
similar-sounding word happened. Choice
town on business. Choice (A) confuses
(C) is not mentioned in the conversation.
busy with the similar-sounding word
Choice (D) repeats the word relax out of
business. Choice (C) confuses downtown
context.
with out of town. Choice (D) associates
calling, which is what the woman is 65. (A) The speakers are discussing a hotel
doing, with phone. reservation and the woman says I don’t
see anything here under your name. Choices
57. (C) is what the woman says she will do.
(B), (C), and (D) are not mentioned.
Choice (A) is what the man suggests she
do. Choice (B) repeats the word return. 66. (C) The man says I need one for three days.
Choice (D) repeats the word call. Choice (A) repeats the word one out of
context. Choice (B) is not mentioned.
58. (A) The woman says she’d like him to look
Choice (D) confuses four with the word
at some contracts. Choice (B) confuses book
for.
with the similar-sounding word look.
Choice (C) confuses bank with the 67. (D) $165 is the price the woman
similar-sounding word thank. Choice (D) mentions. Choices (A), (B), and (C)
confuses a boat with the similar-sounding confuse similar-sounding numbers with
word about. the correct answer.

INTRODUCTORY COURSE ANSWER KEY: PRACTICE TEST ONE 25

Downloaded by Minh Nguy?n V?n (nguyenvanminh07012003@gmail.com)


lOMoARcPSD|9770843

68. (A) The woman mentions Carl’s people. Choices (B) and (C) do not fall
promotion. Choice (B) repeats the word into the low-income or indigent category.
house, but it is the speaker’s house that is Choice (D) is not mentioned.
mentioned, not Carl’s. Choice (C) 76. (C) They are changing the name because
associates got married with wife. Choice they would like to have a separate
(D) repeats the word office, but nothing is identity. Choice (A) is not correct because
mentioned about Carl painting an office. the new name is longer than the old one.
69. (B) The man says that he is happy for Carl. Choice (B) may be true but is not the
Choice (A) confuses unhappy with happy. reason for the name change. Choice (D) is
Choice (C) confuses fearful with the not mentioned.
similar-sounding word feel. Choice (D) 77. (A) The restaurant specializes in seafood.
confuses mad with the similar-sounding Choices (B), (C), and (D) are dishes
word bad. served at the restaurant, but they aren’t
70. (D) The man suggests having a party. mentioned as specialties.
(A) Carl’s wife is mentioned in the 78. (C) The speaker says that the restaurant
conversation, but nothing is said about is full of wonderful aromas and tastes.
meeting her. (B) The office staff is Choice (A) is incorrect because the
mentioned, but leaving the office isn’t. speaker says the dishes are moderately
(C) It is the speaker’s house that is priced. Choice (B), vegetarian dishes, are
mentioned, not Carl’s. mentioned, but meat dishes are
mentioned as well. Choice (D) is
PART 4 (PAGE 236) incorrect because the restaurant serves
71. (D) A smoker started the fire. Choice (A) Japanese food only.
associates park ranger with park. Choices
79. (A) The speaker says that the restaurant
(B) and (C) are mentioned in the talk but
is open seven days a week. Choice (B) is not
not as people responsible for the fire.
correct. Choice (C) confuses you with the
72. (A) The fire destroyed campgrounds in days that breakfast is served. Choice (D)
the park. Choice (B), houses, are confuses you with the days the business
mentioned as threatened by the fire, but lunch is offered.
not destroyed. (C) The smoker’s car is
Copyright © 2012 by Pearson Education, Inc. Permission granted to reproduce for classroom use.

80. (C) They are meeting to elect the


mentioned, but it wasn’t destroyed.
Employee of the Year. Choices (A), (B), and
Choice (D), a town, White River, is
(D) are not correct.
mentioned as being close to the fire, but
not destroyed. 81. (A) Employees will vote by writing the
names of their choice on ballot slips.
73. (C) They expect the fire to be completely
Choice (B), (C), and (D) are not
put out by today. Choice (A) is the day
mentioned.
people had to flee their homes. Choice
(B) is the day the fire was 75 percent 82. (B) The winner will receive two prizes: a
under control. Choice (D) confuses two cash prize and a vacation. Choices (A),
days with the similar-sounding word (C), and (D) are not correct.
today. 83. (A) The tour takes place in the morning.
74. (D) It is a place that provides medical care, Choices (B), (C), and (D) are contradicted
so it is a hospital. Choices (A) and (B) are by the tour guide’s greeting, good
other types of places where people can morning.
receive services. Choice (C) is confused 84. (C) The tour will finish at the main plaza.
with the former name of the place Villa Choice (A) is where the tour starts.
Hospitalis. Choice (B) is the place they are touring.
75. (A) The hospital provides care for low- Choice (D) is what they will see from the
income and indigent people. These are poor main gate.

INTRODUCTORY COURSE ANSWER KEY: PRACTICE TEST ONE 26

Downloaded by Minh Nguy?n V?n (nguyenvanminh07012003@gmail.com)


lOMoARcPSD|9770843

85. (B) The tour guide requests that the dates that sound similar to the correct
tourists all stay close together. Choices (A), answer.
(C), and (D) are not mentioned. 94. (C) For a limited time, two-for-one
86. (A) The speaker says that she is returning memberships are being offered. Choices
a phone call. Choice (B) confuses you (A), (B), and (D) are not mentioned.
with the speaker giving her own address. 95. (D) It is the 10:30 train to Springdale.
Choice (C) is incorrect because it is her Choice (A) is when boarding of the train
address, not her phone number, that Max will begin. Choice (B) confuses you with
wanted her to verify. Choice (D) is the track number. Choice (C) confuses
incorrect because an appointment is seven with the similar-sounding number
never mentioned. ten.
87. (D) The speaker is not sure whether 96. (B) The speaker says Passengers with small
Max’s last name is Amberson or children will be allowed to board first.
Anderson. Choice (A) is incorrect Choice (A) is incorrect because all
because she knows that Max called to passengers must have reservations.
verify her address. Choice (B) confuses Choice (C) repeats the word bags. Choice
you with the speaker giving Max a time (D) repeats the word pets.
to call her back. Choice (C) is incorrect
because the speaker clearly states the 97. (B) According to the speaker, Each
name of her street. passenger is allowed to take only one small
bag on the train. Choice (A) confuses you
88. (C) The speaker says to call her back after with the long list of things passengers are
three this afternoon. Choices (A) and (B) not allowed to take. Choices (C) and (D)
confuse noon with afternoon. Choice (D) are mentioned as things passengers are
confuses after three days with after three not allowed to take.
this afternoon.
98. (A) There are seven lectures in the series.
89. (A) According to the talk, The heavy rains Choices (B), (C), and (D) are numbers
and strong winds . . . tore down power lines. that sound similar to the correct answer.
Choice (B) repeats the word residents.
Choice (C) repeats work crews. Choice (D) 99. (A) Tonight’s speaker will talk about
butterflies. Choices (B), (C), and (D) are
Copyright © 2012 by Pearson Education, Inc. Permission granted to reproduce for classroom use.

repeats Power Company.


mentioned as the topics of other lectures
90. (D) According to the talk, 150,000 in the series.
residents were without electricity.
Choices (A), (B), and (C) are numbers 100. (D) The speaker says that photos will be
that sound similar to the correct answer. on sale at the end of the talk. Choice (A)
confuses you with the topic of the video.
91. (B) The speaker says The western part of Choice (B) confuses you with the photos
the county will be able to turn on their lights of butterflies on display. Choice (C) is
this evening. Choice (A) repeats the words what will happen before the talk.
Power Company. Choice (C) repeats the
word lucky. Choice (D) repeats the word PART 5 (PAGE 239)
county.
101. (B) A noun is needed after the definite
92. (B) The opening of LaForce Fitness article. Deductions, which means “money
Center is being announced. Choice (A) taken out,” is logical here. Choice (A) is a
confuses a golf tournament and golf course verb. Choice (C) is a noun but isn’t
being offered at the center. Choice (C) logical here because it deals with
confuses similar-sounding renovation and insurance. Choice (D) is a gerund but
orientation. Choice (D) is contradicted by isn’t logical here.
free orientation sessions.
102. (A) A past participle is required for the
93. (A) The fitness center will open on passive voice. Choice (B) is a noun.
November 1. Choices (B), (C), and (D) are Choice (C) is the basic verb form. Choice
(D) is the simple present.

INTRODUCTORY COURSE ANSWER KEY: PRACTICE TEST ONE 27

Downloaded by Minh Nguy?n V?n (nguyenvanminh07012003@gmail.com)


lOMoARcPSD|9770843

103. (D) The adverb yet is used with a something for a future result. Choice (B),
negative verb in the present perfect and because, explains the past or current
is always placed after the verb phrase: situation that causes something to
hasn’t arrived yet. Choice (A) is an happen. Choices (C) and (D) are not
affirmative adverb used with present logical in this sentence.
perfect. Choice (B) works the same as yet 111. (C) The adverb surprisingly modifies the
but is always placed before the verb adjective large: surprisingly large. Choice
phrase: still hasn’t arrived. Choice (C) is (A) is the simple past. Choice (B) is an
illogical in this idea. adjective. Choice (D) is a noun.
104. (C) A past participle is required with the 112. (A) The preposition within means “in.” It
auxiliary has to form the present perfect. can be used for concrete things (within the
Choice (A) is the simple present. Choice room; within the city) or with abstract
(B) is the basic verb form. Choice (D) is things (within his job description; within the
the present participle. court’s jurisdiction). Choice (B), into, is a
105. (A) Requested, which means “asked to do preposition of movement and doesn’t
something,” is the only logical choice for work in this case. Choice (C), about, does
this idea. Choice (B), referred, means not make sense in this context. Choice
“directed to a source of help or (D), inside, is mostly used with concrete
information.” Choice (C), rejected, means things: inside the room; inside the city.
“refused, denied.” Choice (D), reported, 113. (B) Realistic is an adjective that means
means “told about.” “concerned with facts as they really are”
106. (D) The noun remarks is needed to go and is appropriate for this sentence. In
with the possessive adjective our. Choice addition, an adjective is required after
(A) is a gerund but isn’t logical here. the predicate isn’t. Choice (A) is an
Choice (B) is an adjective. Choice (C) is adjective but has the wrong meaning for
the simple past. this sentence. It means “not false, not
artificial.” Choices (C) and (D) are
107. (C) The present participle, or –ing,
adverbs.
adjective amazing is needed to describe
the noun advice. Use the present 114. (B) The adjective collaborative modifies
Copyright © 2012 by Pearson Education, Inc. Permission granted to reproduce for classroom use.

participle adjective when describing the noun effort. Choices (A) and (C) are
what causes or creates the feeling: My nouns. Choice (D) is an adverb.
cousin’s amazing advice. Choice (A) is an 115. (D) The possessive pronoun yours is the
adverb. Choice (B) is the simple past. It object of the preposition of. Choice (A) is
can also be the past participle adjective, a possessive adjective. Choice (B) is the
which describes the person or people subject pronoun. Choice (C) is the
receiving the feeling: We were amazed. reflexive pronoun and does not make
Choice (D) is a noun. sense in this context.
108. (B) A preposition is needed: to be surprised 116. (D) Research means “looking into
at. Choices (A), (C), and (D) don’t go with scientific information” and fits the idea
to be surprised. of this sentence. Choice (A), investigation,
109. (D) Refund, which means “cash or credit means “looking into information, usually
received for a return,” is the only logical by the police or government.” Choice (B),
choice for this sentence. Choice (A), investing, means “putting money into a
reimbursement, means “money you paid company.” Choice (C), planning, means
that is repaid.” Choice (B), premium, “deciding how to do something in the
means “an amount you pay for future.”
insurance.” Choice (C), duplication, 117. (D) Prepare means “to make ready” and
means “copying something.” fits the idea of this sentence. Choice (A),
110. (A) So that is a conjunction that explains compensate, means “to make payment.”
the purpose or intention of doing Choice (B), deliver, means “to bring

INTRODUCTORY COURSE ANSWER KEY: PRACTICE TEST ONE 28

Downloaded by Minh Nguy?n V?n (nguyenvanminh07012003@gmail.com)


lOMoARcPSD|9770843

something for business reasons.” Choice subject pronoun. Choice (C) is used with
(C), apply, means “to ask for in a formal the preposition on: on its own. Choice (D)
way, usually in writing.” is an incorrect form.
118. (A) The comparative adjective different is 128. (C) Expand means “to increase, get
always followed by the preposition from. larger,” and fits the idea of this sentence.
Choices (B), (C), and (D) are therefore Choice (A), extrapolate, means “to infer
incorrect. unknown information from known
information.” Choice (B), exhibit, means
119. (B) When a sentence begins with a
“to show.” Choice (D), exhale, means “to
negative adverb of frequency, there is a
breathe out.”
word inversion like the word order for a
yes/no question: Never has there been . . . . 129. (C) Aggressive means “assertive, bold,”
Choices (A), (C), and (D) are not negative and fits the idea of this sentence. Choice
adverbs of frequency and are therefore (A), outrageous, means “disgraceful,
incorrect. offensive, shameless.” Choice (B),
lucrative, means “profitable.” Choice (D),
120. (C) The preposition by means “before or
astounding, means “very amazing,
at, but not later than.” Choices (A), (B),
surprising.”
and (D) are not used in this context with
the phrase the end of the year. 130. (D) A noun is needed after the adjective
every. Intention is the correct choice here
121. (C) The noun accuracy is needed after the
because it means “a plan of action or a
adjective utmost. Choice (A) is an
design” and it fits the idea. Choice (A) is a
adjective. Choice (B) is an adverb. Choice
noun, but its meaning is “a purpose” and
(D) is a quantifying phrase but is not
it doesn’t fit this idea. Choice (B) is an
logical after utmost.
adverb. Choice (C) is a verb.
122. (B) The adjective periodic modifies the
131. (B) With prepositions, we need to use the
noun questionnaires. Choice (A) is a noun.
direct object form of pronouns. Whom is
Choice (C) is an adverb. Choice (D) is
the direct object form. Choice (A) is the
a noun.
subject pronoun. Choice (C) is used for
123. (D) The adverb anxiously modifies the verb things, not people. Choice (D) is the
awaiting. Choice (A) is a noun. Choice (B)
Copyright © 2012 by Pearson Education, Inc. Permission granted to reproduce for classroom use.

incorrect pronoun.
is an adjective. Choice (C) is a noun.
132. (B) Screened means “investigated” and
124. (C) Report means “a document about fits the idea of this sentence. Choice (A),
something” and fits the idea of this screamed, means “shouted, yelled.”
sentence. Choice (A), accounting, means Choice (C), scrawled, means “not written
“managing the books and taxes for clearly.” Choice (D), scraped, means
companies or individuals.” Choice (B), “rubbed with a lot of pressure.”
documentation, means “written proof of
133. (A) Because of the possessive, Mr.
something.” Choice (D), observance,
Hansen’s, the gerund noun resigning must
means “celebrating a holiday or
be used. Choice (B) is the simple past.
tradition.”
Choice (C) is the simple present. Choice
125. (A) On is the preposition used after the (D) is the basic verb form.
verb elaborate. Choices (B), (C), and (D)
134. (C) Predictions means “saying what you
are illogical.
think will be in the future” and fits the
126. (B) It is recommended that requires the idea of this sentence. Choice (A),
basic verb form seek out. Choice (A) is the predicaments, means “trouble, problems.”
present progressive. Choice (C) is Choice (B), predilections, means
the infinitive verb. Choice (D) is the “preferences.” Choice (D), predicates,
simple past. means “the parts of sentences that
127. (A) The reflexive pronoun itself means include the verb and object.”
“alone” in this context. Choice (B) is the

INTRODUCTORY COURSE ANSWER KEY: PRACTICE TEST ONE 29

Downloaded by Minh Nguy?n V?n (nguyenvanminh07012003@gmail.com)


lOMoARcPSD|9770843

135. (B) The present participle, adjective, PART 6 (PAGE 243)


exciting, modifies the noun possibilities.
141. (B) The bank branch will open at a future
Present participle adjectives describe the
date, so a future verb is required when
person or thing creating the feeling.
describing it. Choice (A) is present perfect
Choice (A) is the past participle adjective,
tense. Choice (C) is a present participle.
which describes the person or thing
Choice (D) is simple past tense.
receiving the feeling. Choice (C) is a
noun. Choice (D) is an adjective but 142. (D) There plus to be is followed by a noun
means “easily excited.” or list of nouns, so the noun form
entertainment is correct. Choice (A) is a
136. (D) Unimaginable means “something we
verb. Choice (B) is a noun, but it refers to
couldn’t have imagined before” and fits
a person. Choice (C) is a gerund.
the idea of this sentence. Choice (A),
unacceptable, means “something nobody 143. (C) The announcement is directed at
can agree to or accept.” Choice (B), customers, who usually go to a bank to
implacable, means “cannot be pleased.” open an account. Choices (A), (B), and (D)
Choice (C), inadvertent, means are things that would more likely be
“accidental, unintentional.” opened by bank employees.
137. (A) Events means “important 144. (C) The article mentions computer,
occurrences” and fits the idea of this software, and communications
sentence. Choice (B), reunions, means companies, in other words, technology
“people meeting after a long separation.” companies. Choices (A), (B), and (D) are
Choice (C), councils, means “groups of sectors of the economy but are not
experts called together for consultation.” mentioned in the article.
Choice (D), invocations, means “prayers.” 145. (D) The paragraph talks about a rise in
138. (A) The construction such . . . that . . . is profits, which means the same as earnings.
used to quantify how much of a success Choice (A) is something a company
the blood drive was. Such is always used would not hope to increase. Choices (B)
with a noun phrase: such a success. Choice and (C) are things a company might
(B), so, is used with adjectives or adverbs. increase but are not the subject of the
Choices (C) and (D) do not make sense in paragraph.
Copyright © 2012 by Pearson Education, Inc. Permission granted to reproduce for classroom use.

this context. 146. (B) Costly is an adjective, and an adjective


139. (B) The adjective collective means “acting is required, describing the wireless
as one,” which is appropriate for the idea service. Choice (A) is a noun or a verb.
of this sentence. Choice (A) is the past Choice (C) is a verb form of another word
participle adjective,” and it doesn’t work with a different meaning. Choice (D) is a
in this idea. Choice (C) is a present gerund.
participle adjective. Choice (D) is an 147. (C) The writer is addressing Ms.
adjective that means “something worth Harwood about Ms. Harwood’s staff, so
collecting for its value,” so it does not the second person possessive adjective
make sense in this context. your is the correct form. Choice (A) is the
140. (D) The subordinate conjunction despite is first person singular form. Choice (B) is
used with a direct object. Choice (A) is the first person plural form. Choice (D) is
missing the preposition of. Choice (B) the third person plural form.
does not make sense in this context. 148. (D) An adverb is required here to
Choice (C) requires a noun and verb describe how the job was done. Choice
phrase after it, and it does not make (A) is an adjective. Choice (B) is a past
sense in this context. tense verb. Choice (C) is a gerund.
149. (A) The writer of the letter was a guest at
the hotel, and stay is what hotel guests

INTRODUCTORY COURSE ANSWER KEY: PRACTICE TEST ONE 30

Downloaded by Minh Nguy?n V?n (nguyenvanminh07012003@gmail.com)


lOMoARcPSD|9770843

do. Choice (B) is what a hotel employee 157. (A) It is necessary for all employees to
would do. Choices (C) and (D) are not give up two days’ pay by scheduling the
important activities at a hotel. two days they do not report to work.
Choice (B) confuses June 30 with the date
150. (B) To renew a contract means to extend
by which the budget must be balanced.
it or keep it in effect for more time. The
Choice (C) is contradicted by the plan
notice says the contract will expire soon,
announced. Choice (D) is contradicted by
so renewing the contract would prevent
having to balance the budget by this
it from expiring. Choice (A) is just one
fiscal year.
part of making a new contract. Choice (C)
means to stop the contract. Choice (D) 158. (D) The two left lanes on the north side
means to read the contract. will be closed. Choices (A), (B), and (C)
are contradicted by the two left lanes on
151. (A) This is an imperative verb giving an
the north side will be closed.
instruction. Choice (B) is an infinitive
verb. Choice (C) is a gerund. Choice (D) 159. (B) Miami-Dade County, the issuer of the
is a noun. notice, is authorizing the lane closures.
Choices (A), (C), and (D) are not
152. (A) Until means before. The contract will
mentioned.
not be effective before payment is
received; then when the payment is 160. (C) The NE 151st St. exit falls outside of
received, it will become effective. the NE 79th St. to NE 135th St. area that
Choices (B) and (C) would make the will be affected. Choices (A), (B), and (D)
sentence mean the contract will not be fall within the area that will be affected.
effective after payment is received, which 161. (D) The article is about how much money
doesn’t make sense. Choice (D) means at old covers of Movie Goer Magazine are
the same time, which also doesn’t make worth. Choice (A) confuses literary value
sense. with monetary value. Choice (B) is
incorrect because although Mr. Boulis is
PART 7 (PAGE 247) mentioned, he is not the main subject of
153. (B) The announcement talks about relaxing, the article. Choice (C) confuses issues
full bodywork, and chair for neck, shoulders, with covers.
and back; the announcement is about
Copyright © 2012 by Pearson Education, Inc. Permission granted to reproduce for classroom use.

162. (B) The writer assumes readers have old


massages. Choice (A) confuses length of
issues in the basement. Choices (A), (C),
breaks with needing a real break. Choice (C)
and (D) are not mentioned.
associates furniture with table and chair.
Choice (D) confuses similar sounds 163. (A) The cover of the first issue, the
language and lounge. premier issue, of the magazine is the
most valuable. Choice (B) is the opposite
154. (B) The service is free of charge for
of the first one. Choices (C) and (D) are
employees. Choice (A) confuses $15 for
valued at less than the value of the first
15 minutes and the 15 minutes the
one.
massage takes. Choice (C) is not
mentioned. Choice (D) confuses Human 164. (B) Financial Success is on the air for one
Resources having the rates and Human hour, from 8:00 to 9:00. Choices (A), (C),
Resources offering the service. and (D) are incorrect.

155. (C) Employees must give up two days of 165. (C) There are six different business shows
pay to end the budget crisis. Choices (A), being televised; one is being shown
(B), and (D) are not mentioned. twice. Choices (A), (B), and (D) are
incorrect.
156. (B) Because of poor profits, the budget is
short by about $13 million. Choice (A) is 166. (A) Business Week in Review is being
the opposite of a shortage. Choices (C) repeated. Choices (B), (C), and (D) are not
and (D) are contradicted by poor profits financial/business shows.
causing a shortage.

INTRODUCTORY COURSE ANSWER KEY: PRACTICE TEST ONE 31

Downloaded by Minh Nguy?n V?n (nguyenvanminh07012003@gmail.com)


lOMoARcPSD|9770843

167. (B) BPL is the only station that shows only to the charger device. Choice (D) repeats
business programs. Choices (A) and (C) the words normal operating: this is not one
show different types of programs. Choice of the steps.
(D) shows only movies. 178. (B) If the device fails to charge properly,
168. (C) A department within an organization, contact your dealer. Choices (A), (C), and
such as a police department, is often (D) are not mentioned.
called a unit. The other options do not fit 179. (D) You can study at home in your spare
the context. time. Choices (A), (B), and (C) are times
169. (B) Michael O’Brien was freed by a group you can study if those are considered
of judges in the Court of Appeals. Choice your individual spare times.
(A) associates the Queen with Great 180. (D) You could get an accounting degree
Britain. Choice (C) is who is deciding on in as little as two years. Choice (A) is
his compensation. Choice (D) is not how long it takes to get a career degree as
mentioned. a private investigator or medical
170. (C) Mr. O’Brien was convicted of transcriptionist. Choice (B) is how long it
industrial espionage. Choices (A), (B), takes to get a high school diploma.
and (D) are crimes but not the one he was Choice (C) confuses similar sounds 18
convicted for. months and 8,000,000 men and women.
171. (D) To commit a crime is to carry out a 181. (C) The first e-mail says that it is a
crime. Options (A), (B), and(C) do not fit department meeting, so department members
the context. will attend. Choice (A) confuses you with
the location of the meeting (conference
172. (A) Mr. O’Brien is upset because he has to
room). Choice (B) is incorrect because the
reimburse the prison for room and board.
meeting is only for department members.
Choice (B) confuses paying money to the
Choice (D) is the person with whom Mr.
Home Office and the Home Office ruling
Richards will meet on Friday.
that he would have to reimburse the prison.
Choices (C) and (D) are not mentioned. 182. (A) The e-mail says that the meeting will
take place in Conference Room 2. Choices
173. (D) There are four ways to buy
(B) and (C) confuse you with the lunch
merchandise at “Trendy Male”: at their
Copyright © 2012 by Pearson Education, Inc. Permission granted to reproduce for classroom use.

that will be served at the meeting. Choice


stores, by phone, by mail, and online.
(D) is the original location of the meeting,
Choices (A), (B), and (C) are incorrect.
but according to the e-mail, that location
174. (B) If you buy an item on sale, the second was changed.
item will be 50% off. You would pay $23
183. (B) Thursday is the day Mr. Richards will
for the second shirt if you buy the first
leave for Sydney. Choice (A) is the day
for $46. Choices (A), (C), and (D) are not
people must notify Ms. Santelli if they
correct.
can’t attend the meeting. Choice (C) is
175. (C) The offer lasts 21 days, from June 17 the day Mr. Richards will meet with Mr.
though July 7. Choices (A), (B), and (D) Lockerman. Choice (D) is the day Mr.
are contradicted by the dates given. Richards will send his report about his
176. (C) These instructions are for a cellular meeting with Mr. Lockerman.
phone. Choice (A) confuses portable phone 184. (D) In his e-mail, Mr. Richards mentions
with cellular phone. Choices (B) and (D) that Peter worked with me on this, in
are associated with the technical terms reference to his report. By looking at the
mentioned in the article but are incorrect. addressees of the first e-mail, we can see
177. (A) If the battery is not charging, check that Peter’s last name is Kim. Choices (A),
that it is connected to an approved (B), and (C) are the other addressees of
charging device. Choice (B) is not the first e-mail, but none of them is
mentioned. Choice (C) is what you named Peter.
should do after you check the connection

INTRODUCTORY COURSE ANSWER KEY: PRACTICE TEST ONE 32

Downloaded by Minh Nguy?n V?n (nguyenvanminh07012003@gmail.com)


lOMoARcPSD|9770843

185. (A) Mr. Richards’ e-mail says he will 194. (D) Ms. Heinz wants to travel business
leave figures you’ll need for the second item class. A one-way ticket is $175, so a
on the meeting agenda. Choice (B) is the round-trip ticket is $350. Choice (A) is the
person with whom Mr. Richards will price for a one-way tourist class ticket.
meet in Sydney, but nothing is mentioned Choice (B) is the price for a one-way
about his address. Choices (C) and (D) business class ticket. Choice (C) is the
are mentioned in the e-mail but are not price for a round-trip tourist class ticket.
the correct answer. 195. (D) Ms. Heinz says that she wants to
186. (A) In this letter, H. J. Simpson says that return about 5 or 6 in the afternoon.
he received the shipment on November Choices (A), (B), and (C) are not
18. It was shipped on November 15. mentioned.
Choice (B) is based on the date the letter 196. (A) The ad says to contact Cathy Chang,
was written. Choice (C) is not mentioned. Events Manager, and the e-mail is
Choice (D) is not mentioned. addressed to Cathy Chang. Choice (B) is
187. (A) The letter states the items we received the name of the hotel’s Head Manager.
were not the same as the items we ordered. Choice (C) is the name of the person who
Choices (B), (C), and (D) are not wants to host a banquet, and choice (D) is
mentioned. her assistant.
188. (B) The letter states that the model D is 197. (B) The e-mail states I am planning the
half the price of the model B. The invoice annual employee appreciation banquet for my
shows that the model B costs $150, so the company. Choices (A), (C), and (D) are
model D must cost $75. Choice (A) is not other reasons a company might have a
mentioned. Choice (C) is the price of the banquet but are not the correct answer.
model B, not D, shown on the invoice. 198. (D) The e-mail states Our banquet is
Choice (D) is double, not half, the price of scheduled for the evening of July 15. Choice
the model B. (A) looks similar to May 23, the date the
189. (B) The letter states At least the watercooler ad appeared in the paper. Choice (B) is
was correct. By looking at the invoice, we the date the ad appeared in the paper.
see that one watercooler was shipped. Choice (C) July 5 looks similar to July 15.
Copyright © 2012 by Pearson Education, Inc. Permission granted to reproduce for classroom use.

Choices (A), (C), and (D) are not 199. (B) The ad says that the hotel has room
mentioned. for 250 guests, and the e-mail states We
190. (D) The letter states that when he tried only plan to have half the number of guests
calling, the line is always busy. Choices mentioned in your ad. Choice (A) is not
(A), (B), and (C) are possible outcomes mentioned. Choice (C) is the number of
but are not the correct answer. guests mentioned in the ad. Choice (D) is
double, not half, the number of guests
191. (D) The earliest train arrives at 10:30 in
mentioned in the ad.
the morning. Choices (A), (B), and (C)
refer to times listed on the schedule in 200. (D) The banquet planner mentions that
bold, meaning that they are afternoon she wants both vegetarian and meat
and evening arrival times. choices. Choice (A) is incorrect because
she wants vegetarian food as well as
192. (B) The trip lasts four hours and twenty
meat. Choice (B) is incorrect because she
minutes. Choice (C) is the length of the
says I think a five-course dinner would be
trip when intermediate stops are made.
too much. Choice (C) is incorrect because
Choices (A) and (D) are not mentioned.
the banquet is planned for the evening,
193. (C) This is the second train on Tuesday which would mean dinner, not lunch.
morning. Choice (A) is the first train.
Choice (B) is the second train on Monday,
Wednesday, and Friday only. Choice (D)
is a later train and also leaves only on
Monday, Wednesday, and Friday.

INTRODUCTORY COURSE ANSWER KEY: PRACTICE TEST ONE 33

Downloaded by Minh Nguy?n V?n (nguyenvanminh07012003@gmail.com)


lOMoARcPSD|9770843

ANSWER KEY
PRACTICE TEST TWO
PART 1 (PAGE 264) 6. (C) The people are standing at the bottom
of an escalator, talking together. Choice
1. (B) The men are greeting one another.
(A) confuses similar-sounding words
Choice (A) tries to confuse you by using
escalator and elevator. Choice (B) confuses
the similar-sounding phrase shaking out the
similar-sounding words escalator and
sand for shaking the man’s hand. Choice (C)
refrigerator. Choice (D) associates escalator
tries to confuse you by using sea for see.
with stairs and confuses similar-sounding
Choice (D) tries to confuse you by using
words talking and walking.
the similar-sounding phrase putting on a
smile and running a mile. 7. (D) The chairs and tables are empty.
Choice (A) provides incorrect details: the
2. (D) The woman has the file cabinet drawer
tables are empty. Choice (B) tries to
open and is looking in the files. Choice (A)
confuse you by using the related word
tries to confuse you by using the similar-
diners and provides incorrect details:
sounding word drawing for drawer. Choice
there are no diners. Choice (C) tries to
(B) tries to confuse you by using the
confuse you by using the related word
similar-sounding word tiles for files.
customers and provides incorrect details:
Choice (C) uses the word filing in a
there are no customers.
different context. The woman may be
filing (putting in order) documents, but 8. (C) The goods are in cartons stacked in the
she’s not filing (smoothing) her nails. warehouse. Choice (A) tries to confuse
you by using the similar-sounding and
3. (A) The telephones are on the wall. Choice
related word cargo for cartons. Choice (B)
(B) gives incorrect details: the signs are
tries to confuse you by using the related
above the phones. Choice (C) gives incorrect
words boxes and container. Choice (D)
details: the door is to the left of the phones.
provides incorrect details: we do not
Copyright © 2012 by Pearson Education, Inc. Permission granted to reproduce for classroom use.

Choice (D) provides incorrect details: the


know if there is produce in the boxes and
phones are to the right of the door.
the boxes are on the front of a forklift, not
4. (A) A woman in a laboratory is writing on the back of a truck.
some notes. Choice (B) mentions the
9. (D) They are getting on the train. Choice
microscopes in the photo, but the woman is
(A) tries to confuse you by using the
not looking through them. Choice (C)
related word platform. Choice (B) tries to
mentions the lab equipment in the photo,
confuse you by using the related word
but the woman is not cleaning it. Choice
window; there are windows on the train,
(D) confuses telescope with microscope.
but they’re not being opened. Choice (C)
5. (B) The gate to the courtyard is open. tries to confuse you by using the similar-
Choice (A) provides incorrect details: sounding word plane for train.
there is no guard in the picture; and tries
10. (D) The woman is pouring coffee into the
to confuse you by using the related word
cup. Choice (A) provides incorrect
entrance. Choice (C) provides incorrect
details: she’s holding the cup with one
details: the pathway is straight, but the
hand and pouring the coffeepot with the
entrance is circular. Choice (D) tries to
other. Choice (B) uses the word pouring in
confuse you by using the related word
a different context. Choice (C) tries to
garden and provides incorrect details:
confuse you by using the similar-
there are no tourists lined up at the door.
sounding word pointing for pouring.

INTRODUCTORY COURSE ANSWER KEY: PRACTICE TEST TWO 34

Downloaded by Minh Nguy?n V?n (nguyenvanminh07012003@gmail.com)


lOMoARcPSD|9770843

PART 2 (PAGE 270) 23. (A) You don’t look sick responds to the
speaker’s not feeling well. Choice (B) uses
11. (C) Late last night answers when. Choice
the similar-meaning word fine for well.
(A) answers how long. Choice (B) answers
Choice (C) uses the similar sounding
where.
welcome for well.
12. (B) Mr. Kim’s answers whose? Choice (A)
24. (B) That restaurant across the street
answers where. Choice (C) confuses
answers where. Choice (A) answers when.
similar sounds they’re keys and these are.
Choice (C) answers would you like to eat.
13. (B) On her desk answers where. Choice (A)
25. (B) Yes, I got a room at a nice place
confuses where is Ms. Sato and where are
downtown answers were you able to book a
her packages. Choice (C) confuses similar
hotel. Choice (A) confuses the similar
sounds packed and packages.
words book (n.) and book (v.). Choice (C)
14. (C) Yes, I just have to finish typing these answers did you book a hotel.
notes answers the yes/no question. Choice
26. (B) Ask her if she wants to sit down and wait
(A) uses the similar-sounding word met.
addresses the topic of someone who has
Choice (B) answers where.
been waiting for an hour. Choice (A) tries
15. (B) In the garage answers where do you to confuse you by using the time word
usually park. Choice (A) confuses park (n.) clock for an hour. Choice (C) uses weight
and park (v.). Choice (C) associates for wait.
mechanic with car.
27. (C) You should speak with her assistant
16. (A) Tomorrow morning answers when. answers how can I make an appointment.
Choice (B) answers what can I call you. Choice (A) associates position and
Choice (C) associates talk and call and appointment. Choice (B) confuses similar
answers where. sounds disappointed and appointment.
17. (B) I’d better bring an umbrella responds to 28. (A) Only about 3 or 4 days answers how
the idea It’s supposed to rain tomorrow. long. Choice (B) answers how long has it
Choice (A) repeats the word supposed and been since you were last there. Choice (C)
uses the associated words yesterday for answers where will you stay.
tomorrow. Choice (C) repeats the word
29. (B) Yes, that’s a great idea answers would
Copyright © 2012 by Pearson Education, Inc. Permission granted to reproduce for classroom use.

tomorrow and confuses the similar-


you like to go. Choice (A) confuses similar
sounding word train for rain.
sounds walk and work. Choice (C) repeats
18. (B) Mrs. Garcia answers who. Choice (A) the word worked.
answers where is Mr. Contini. Choice (C)
30. (B) Yes, she was there answers did you notice
answers when.
whether Ms. Kovacs was at the reception.
19. (A) Everyone leaves by 5:30, so the office is Choice (A) confuses similar sounds notes
considered closed by then. Choice (B) and notice. Choice (C) confuses similar
repeats the offices are closed. Choice (C) sounds received and reception.
confuses similar sounds clothes and close.
31. (A) In his office answers where. Choices (B)
20. (C) Because we have to discuss the budget and (C) answer when.
answers why. Choices (A) and (B) answer
32. (B) There are 15 answers how many. Choice
when.
(A) confuses similar sounds apartment and
21. (B) Once every month answers how often. department. Choice (C) answers how long.
Choice (A) answers who. Choice (C)
33. (C) Next Monday answers when. Choice
confuses similar sounds reporter and
(A) associates cafeteria with lunch. Choice
reports.
(B) associates cost with free.
22. (A) About 15 minutes answers how long.
Choice (B) answers how big. Choice (C)
answers how is the ride.

INTRODUCTORY COURSE ANSWER KEY: PRACTICE TEST TWO 35

Downloaded by Minh Nguy?n V?n (nguyenvanminh07012003@gmail.com)


lOMoARcPSD|9770843

34. (B) Yes, at the newsstand answers did you 44. (D) They have just finished lunch at a
buy that newspaper downstairs. Choice (A) restaurant. Choice (A) is not
repeats the word paper. Choice (C) mentioned. Choice (B) associates bank
confuses similar sounds good-bye and buy. with pay. Choice (C) is where the man’s
wallet is.
35. (B) Next week answers when. Choice (A)
answers where. Choice (C) describes the 45. (B) The woman will pay for lunch with
new assistant manager. her credit card. Choice (A) confuses card
with credit card. Choice (C) confuses play
36. (C) I can recommend an architect responds
with the similar-sounding word pay.
with the type of occupation involved in
Choice (D) associates cook a meal with
building a home. Choice (A) repeats the
lunch.
word building and uses the similar word
home for house. It also repeats the word 46. (A) The man says that he will pay the
new. Choice (B) repeats the verb going. woman back. Choice (B) confuses letters
with the similar-sounding word later.
37. (A) 25% off the regular price answers is
Choice (C) is what the man left at home.
there a discount. Choice (B) confuses
Choice (D) associates meal with lunch.
similar sounds counted and discount.
Choice (C) repeats the word furniture. 47. (B) The man wants dinner reservations
for three people. Choice (A) is not
38. (B) Because he hasn’t had a chance to
mentioned. Choice (C) confuses eight
answers why. Choice (A) confuses similar
with the time 8:45. Choice (D) confuses
sounds newspaper and papers. Choice (C)
you with the time they will eat dinner.
answers has he signed those papers.
48. (C) The man says that he will take the
39. (B) It’s time to cut it responds to the topic
9:00 table. Choice (A) is not mentioned.
of the high grass. Choice (A) uses the
Choice (B) is another option the woman
similar word taller for higher and repeats
offered. Choice (D) confuses ten with the
the word than. Choice (C) uses the similar
similar-sounding word then.
sounding word glass for grass and never
for ever. 49. (C) The man says that they will sit in the
bar until 9:00. Choice (A) repeats the
40. (A) No, I got a new job answers you aren’t
word kitchen. Choice (B) repeats the word
still working for the same company. Choice
Copyright © 2012 by Pearson Education, Inc. Permission granted to reproduce for classroom use.

table. Choice (D) confuses car with the


(B) does not answer the question. Choice
similar-sounding word bar.
(C) confuses similar sounds computer and
company. 50. (C) The speakers are discussing putting
colors on the walls and windows. Choice
PART 3 (PAGE 271) (A) windows is mentioned, but washing
41. (D) The woman says that the secretary them is not. Choice (B) chairs is
sent the package. Choice (A) is incorrect mentioned, but buying them is not.
because the package is for Mr. Ozawa, Choice (D) the rug is mentioned, but
not from him. Choice (B) is the person cleaning it is not.
with whom Mr. Ozawa will meet 51. (D) The woman says that blue would
tomorrow. Choice (C) is mentioned as a match the color of the rug. Choice (A)
person who did not send the package. confuses green with the similar-sounding
42. (D) The meeting is tomorrow. Choices (A) word seems. Choices (B) and (C) are
and (C) are not mentioned. Choice (B) is possible colors the woman mentions for
when the package arrived. the walls.

43. (A) The man says that Mr. Ozawa is at 52. (C) The woman says they will start work
lunch. Choice (B) is where the man will on Friday. Choices (A), (B), and (D) are
put the package. Choice (C) is where Mr. not mentioned.
Ozawa will go tomorrow. Choice (D)
associates mailroom with mail.

INTRODUCTORY COURSE ANSWER KEY: PRACTICE TEST TWO 36

Downloaded by Minh Nguy?n V?n (nguyenvanminh07012003@gmail.com)


lOMoARcPSD|9770843

53. (B) Jim missed the memo because he had 61. (B) The woman will look for the boss in
to type his report. Choices (A) and (C) are his office after lunch. Choice (A) is when
not mentioned. Choice (D) confuses copy she will take the boss to dinner. Choice
machine with final copy. (C) repeats the word lunch. Choice (D)
repeats the word dinner.
54. (A) It was a 3:00 meeting. Choice (B) is
not mentioned. Choice (C) is the time of 62. (D) He is talking about a sale on
the man’s train. Choice (D) confuses eight briefcases. Choice (A) refers to the
with the similar-sounding word late. woman’s suggestion that he fax his order
to the store. Choice (B) is where he read
55. (D) The man is afraid that he will miss
the ad. Choice (C) is how he will make
his train. Choice (A) repeats the word
his order.
meeting, but the meeting is already over.
Choice (B) confuses letter with the 63. (B) The briefcases are 15% off. Choice (A)
associated word memo. Choice (C) confuses ten with the similar-sounding
confuses rain with the word train. word then. Choice (C) is not mentioned.
Choice (D) confuses 50 with the similar-
56. (A) The woman suggests going to the
sounding number 15.
conference room and the man agrees.
Choice (B) confuses business office with 64. (C) The man says that Saturday is the last
noisy office. Choice (C) is how they will day of the sale. Choice (A) is when the
get to the conference room. Choice (D) is woman suggests making the order.
where they are now, but they can’t work Choice (B) is when the woman believes
there because of the noise. the sale is over. Choice (D) confuses next
week with on sale this week.
57. (C) The man suggests taking a computer
with them. Choices (A) and (B) are the 65. (B) The copies are coming out too light
things that the woman says they don’t because the photocopier is broken.
need. Choice (D) associates computer Choice (A) confuses similar-sounding
paper with computer, but they are taking a come and conditioner. Choice (C)
folder of unspecified paper. associates telephone with call. Choice (D)
uses the word light in a different context.
58. (D) The man says We can’t write this report
here and wants to find another place to do 66. (A) The woman says that the repair
Copyright © 2012 by Pearson Education, Inc. Permission granted to reproduce for classroom use.

the work. Choice (A) associates pens and person will be here at noon. Choice (B)
notepads with office supplies. Choice (B) confuses week with the similar-sounding
confuses planning a conference with the word work. Choice (C) is when the man
conference room. Choice (C) confuses fixing wants to report the problem. Choice (D)
a computer with the computer the confuses afternoon with noon.
speakers are using for their work. 67. (B) The man says that he will read his e-
59. (B) The man, Sam, says that his boss will mail. Choice (A) confuses eat a meal with
be in the office tomorrow. Choice (A) is the similar-sounding word e-mail. Choice
incorrect because Sam says that he will (C) uses the word report in a different
be at a conference. Choice (C) is not context. Choice (D) is what the woman
mentioned. Choice (D) is mentioned but already did.
as not being helpful. 68. (C) The man can’t meet this week
60. (C) The woman says that she needs because he will be away on a business
someone to go over accounts with her. trip. Choice (A) repeats the word meet.
Choice (A) confuses cooking lunch with Choice (B) confuses rest this week with the
looking for the boss after lunch. Choice rest of this week. Choice (D) repeats the
(B) confuses book with the similar- word office.
sounding word look. Choice (D) confuses 69. (A) The woman says that she wants to
planning a conference with the conference meet to discuss conference plans. Choice
that Sam will attend. (B) is not mentioned. Choice (C) confuses

INTRODUCTORY COURSE ANSWER KEY: PRACTICE TEST TWO 37

Downloaded by Minh Nguy?n V?n (nguyenvanminh07012003@gmail.com)


lOMoARcPSD|9770843

money with the similar-sounding word produce department. Choice (C) associates
Monday. Choice (D) is where the man will restaurant with the food that’s mentioned.
be Monday morning. Choice (D) associates library with check-
out.
70. (D) 1:00 is the time that they agree on.
Choice (A) confuses 9 with the similar- 78. (A) There is a sale on ground beef.
sounding word fine. Choice (B) confuses Choices (B) and (C) repeat items that are
10 with the similar-sounding word then. mentioned in the talk but that are not on
Choice (C) is the woman’s original sale. Choice (D) confuses suits with the
suggestion for a meeting time. similar-sounding word fruit.
79. (D) Shoppers purchasing 15 or fewer
PART 4 (PAGE 274) items can use the express check-out lanes.
71. (C) The train will leave at 10:30. Choice Choices (A), (B), and (C) are possible, but
(A) confuses 10:10 and 10 minutes. Choice only if they have 15 or fewer items.
(B) confuses 10:15 and Gate 15. Choice (D)
80. (B) A professor is talking to his class.
incorrectly assumes it’s 10:30 now, and
Choice (A) is who he is speaking to.
since boarding starts in 10 minutes, the
Choice (C) associates author with textbook.
train would leave at 10:40.
Choice (D) associates medical doctor with
72. (D) Passengers with small children Dr.
should arrive five minutes before
81. (A) The subject of the class is algebra.
boarding time. Choice (A) confuses offer
Choice (B) confuses similar sounds
help and needing special assistance. Choice
computers and Compton. Choice (C)
(B) is contradicted by passengers who still
confuses similar-sounding finance and
wish to check luggage should do so now.
advanced. Choice (D) associates health
Choice (C) is not mentioned.
with Dr., exercises, and exams.
73. (A) The speaker says that only cash is
82. (B) There will be two tests: a midterm and
accepted on board the train. Choices (B)
a final. Choice (A) is how many textbooks
and (C) are mentioned as unacceptable
they will use. Choice (C) is what time the
forms of payment. Choice (D) uses the
class starts. Choice (D) is the number of
word order out of context.
exercises that will be assigned each week.
Copyright © 2012 by Pearson Education, Inc. Permission granted to reproduce for classroom use.

74. (B) A weather forecaster is talking.


83. (A) Bob Wilson was the city’s first mayor.
Choice (A) associates news reporter with
Choice (B) is who is keeping time. Choice
good news. Choice (C) associates pilot with
(C) confuses war hero with an important
skies. Choice (D) associates travel agent
figure in our history and the war heroes
with trip to the beach.
monument. Choice (D) associates artist
75. (C) The speaker announces the 5:00 weather and art museum.
report and mentions the long day of rain; it
84. (B) The speaker says We’ll have an hour to
must be in the evening. Choice (A) is
spend. Choice (A) confuses half an hour
contradicted by long day of rain. Choice (B)
with the similar-sounding have an hour.
confuses similar-sounding noon and news.
Choice (C) confuses the number 10 with
Choice (D) is when the rain will stop.
the similar-sounding word spend. Choice
76. (C) The speaker says By tomorrow (D) confuses two with to.
morning. . . it’ll be warm and sunny all day.
85. (C) They will see the war heroes
Choice (A) is the weather today. Choice
monument after visiting the Wilson
(B) confuses you with not a cloud in the
House. Choice (A) is where they went
sky. Choice (D) confuses cold with the
before the Wilson House. Choice (B)
similar-sounding word cloud.
confuses bus station and bus driver. Choice
77. (A) You would hear this announcement at (D) is where they will go another day.
a grocery store. Choice (B) confuses
86. (C) The tickets are for the year-end soccer
department store with meat department and
tournament. Choice (A) confuses similar

INTRODUCTORY COURSE ANSWER KEY: PRACTICE TEST TWO 38

Downloaded by Minh Nguy?n V?n (nguyenvanminh07012003@gmail.com)


lOMoARcPSD|9770843

sounds tour with tournament and bus tour the London flight. Choice (C) is confused
with bus ride. Choice (B) confuses similar with the sale on tickets. Choice (D) is
sounds theater and there are. Choice (D) confused with the schedule change next
associates awards ceremony with trophy. month.
87. (B) Tickets can be reserved at the front 97. (C) The message says that the sale ends
desk. Choices (A), (C), and (D) are not next week. Choice (A) is confused with
mentioned as places to get tickets. this evening’s arrivals. Choice (B) confuses
the similar-sounding on the weekend and
88. (B) The bus leaves at 6:00 A.M. Choices
ends next week. Choice (D) is when the
(A) and (C) confuse three with the
schedule will change.
similar-sounding word free. Choice (D)
confuses 6:00 P.M. with 6:00 A.M. 98. (D) The message announces shows at 1, 3,
5, 7:30, and midnight. Choices (A) and
89. (C) You, Too, Can Become a Millionaire is
(C) are confused with show times. Choice
most likely about making money. Choice
(B) confuses two with the similar-
(A) is what Ms. Park is doing now. Choice
sounding word today.
(B) is not likely. Choice (D) confuses retail
business with selling books. 99. (C) The message says All shows before five
o’clock are half price. Choices (A) and (D)
90. (C) Ms. Park will read excerpts from her
are mentioned but not at half price.
book. Choice (A) is what the speaker is
Choice (B) is confused with people who
doing. Choice (B) confuses receive an
will not be admitted to the midnight
award and award-winning author. Choice
show.
(D) confuses make copies and sign copies.
100. (D) This is the only item mentioned as
91. (A) You can purchase the book at the
not allowed. Choices (A), (B), and (C) are
back of the auditorium. Choice (B)
things sold inside the theater.
confuses similar words signs (n.) and sign
(v.). Choice (C) confuses You, Too, Can
PART 5 (PAGE 277)
Become a Millionaire with a game. Choice
(D) is what Ms. Park will do after 101. (D) The noun is needed as the subject of
reading. the sentence. Choice (A) is the past
participle adjective. Choice (B) is the
92. (D) Pamela Jones is the science reporter at
Copyright © 2012 by Pearson Education, Inc. Permission granted to reproduce for classroom use.

present participle adjective. Choice (C) is


the Daily Times newspaper. Choice (A)
the basic verb form.
associates telephone operator with the
phone message. Choice (B) confuses 102. (C) The past participle adjective is
scientist and science reporter. Choice (C) required to describe the subject, which is
confuses an assistant and my assistant. receiving the feeling. Choice (A) is the
present participle adjective, which
93. (A) She can’t answer the phone because
describes the person or thing creating the
she is at a conference. Choices (B), (C),
feeling. Choice (B) is the simple present.
and (D) are contradicted by out of the
Choice (D) is the basic verb form.
office all week at the journalists’ conference.
103. (D) Yet is a negative adverb with present
94. (C) The announcement says To leave a
perfect. It can be placed after the first
message, stay on the line. Choices (A) and
auxiliary (haven’t yet been made) or at the
(B) are mentioned but are contradicted by
end of the sentence (haven’t been made
to leave a message, stay on the line. Choice
yet). Choice (A) is affirmative, so it won’t
(D) is not mentioned.
work. Choice (B) can be used in a
95. (B) The flight from Sydney will arrive on negative present perfect, but it must be
time. Choices (A), (C), and (D) are flights placed before the verb phrase: still haven’t
which are delayed or canceled. been made. Choice (C) does not make
96. (B) The flight from Paris was canceled sense in this context.
because of a blizzard. Choice (A) is 104. (B) The past participle is required to
confused with the reason for the delay of make a present perfect verb form. Choice

INTRODUCTORY COURSE ANSWER KEY: PRACTICE TEST TWO 39

Downloaded by Minh Nguy?n V?n (nguyenvanminh07012003@gmail.com)


lOMoARcPSD|9770843

(A) is the present participle. Choice (C) is “during” with time phrases and does not
the basic verb form. Choice (D) is the make sense in this context. Choice (C),
simple present. later, could work with the preposition
than. Choice (D), above, does not make
105. (A) Admitted means “allowed to enter”
sense in this context.
and fits the idea of this sentence. Choice
(B), omitted, means “left or taken out.” 113. (C) The adjective is required in this
Choice (C), permitted, means “allowed” context to describe the reports. Choice
and does not make sense in this context. (A) is the simple present. Choice (B) is
Choice (D), submitted, means “given to.” the noun. Choice (D) is the basic verb
form.
106. (D) The noun meaning the act of serving is
required to fit the idea of this sentence. 114. (A) The noun is required as the subject of
Choice (A) is a noun meaning people who this sentence. Choice (B) is the simple
serve. Choice (B) is the gerund. Choice (C) past. Choice (C) is the adjective. Choice
is the basic verb form. (D) is the simple present.
107. (B) The present participle adjective is 115. (B) The object pronoun is required.
required to describe the thing that is Choice (A) is the subject pronoun. Choice
creating the feeling. Choice (A) is the (C) is the possessive adjective. Choice (D)
simple present. Choice (C) is the adverb. is the possessive pronoun.
Choice (D) is the simple past or past 116. (A) Current means “present, now” and
participle adjective, which describes the fits the idea of this sentence. Choice (B),
person receiving the feeling. abundant, means “a very large amount”
108. (B) The preposition always used with the and does not make sense in this context.
phrase to be interested is in. Choices (A), Choice (C), actual, means “real.” Choice
(C), and (D) are the wrong prepositions. (D), eventual, means “something that will
happen at some unspecific time in the
109. (C) Crops (vegetables or fruit) are raised
future.”
(cultivated). Choice (A), grown up, means
“become adult” and is used for children 117. (D) Assist means “to help,” and fits the
or young animals. Choice (B), increased, idea of this sentence. Choice (A), resist,
means “made or become more.” Choice means “to work against” and does not
Copyright © 2012 by Pearson Education, Inc. Permission granted to reproduce for classroom use.

(D), enlarged, means “made larger, make sense in this context. Choice (B),
bigger.” desist, means “to stop” or “to cease,”
Choice (C), insist, means “to demand.”
110. (A) The subordinate conjunction although
is used when the idea of the sentence is 118. (D) As . . . as is a comparative phrase
the opposite of what you would expect to showing that two things are equal. The
hear or read, and it fits this idea. Choice adverb well is used because the sentence
(B), because, explains why something has discusses how something acts: as well as.
happened and does not make sense in Choice (A) does not contain the second
this context. Choice (C), so, works like as. Choice (B) uses the wrong
because. In addition, it cannot be used at comparative. Choice (C) uses an
the beginning of the sentence with this adjective instead of an adverb.
meaning. Choice (D), in spite of, requires a 119. (C) A negative adverb of frequency at the
gerund phrase after it because of the beginning of a sentence causes a word
preposition: In spite of her working . . . . inversion to follow: have market conditions
111. (B) An adverb is used to describe an been. Choice (A), reliably, means “able to
adjective. Choice (A) is a past participle depend on” and does not make sense in
adjective. Choice (C) is the basic verb this context. Choices (B) and (D) cannot
form. Choice (D) is the simple present. create the word inversion in this
sentence.
112. (A) Past means “after” and fits the idea of
this sentence. Choice (B), over, means

INTRODUCTORY COURSE ANSWER KEY: PRACTICE TEST TWO 40

Downloaded by Minh Nguy?n V?n (nguyenvanminh07012003@gmail.com)


lOMoARcPSD|9770843

120. (A) The preposition of always follows works.” Choice (D), expressions, means
most if an optional preposition is used. “communicating an idea, especially by
This rule applies to many as well. Choices words.”
(B), (C), and (D) are incorrect 129. (D) Removed means “taken out” or “taken
prepositions. away” and fits the idea of this sentence.
121. (D) The adverb is required to describe Choice (A), reminded, means “made
how the action should be done. Choice somebody remember.” Choice (B),
(A) is the simple past. Choice (B) is the remarked, means “said.” Choice (C),
basic form of the verb. Choice (C) is remodeled, means “changed the interior
the noun. design, usually of a house.”
122. (B) The adjective is required to describe 130. (A) An adjective is required to describe
the subject. Choice (A) is the noun and the noun. Admirable means “very good”
does not make sense in this context. and fits the idea of this sentence. Choice
Choice (C) is an adverb. Choice (D) is a (B) is the past participle adjective
verb. describing the person or thing receiving
the feeling and does not fit the idea here.
123. (A) The noun is required with the
Choice (C) is an adverb. Choice (D) is a
comparative adjective. Choice (B) is the
noun meaning “an officer in the navy.”
present participle adjective. Choice (C) is
the past participle adjective. Choice (D) is 131. (A) The object pronoun referring to a
the adverb. person is required after the preposition.
Choice (B), which, refers to things, not
124. (C) The preposition throughout means “all
people. Choice (C), who, is the subject
over,” “in all parts of,” and fits the idea
pronoun. Choice (D), where, does not
of this sentence. Choice (A) is an
make sense in this context.
adjective. Choice (B), during, is used only
with nouns that refer to periods of time 132. (D) Opinion means “personal thoughts or
(our vacation; the war; the night). Choice feelings” and fits into the idea of this
(D) is an adjective. sentence. Choice (A), reaction, could work
if followed by the preposition to. Choice
125. (C) Against is a negative preposition
(B), mind, does not make sense in this
that fits in with the idea of this sentence.
Copyright © 2012 by Pearson Education, Inc. Permission granted to reproduce for classroom use.

context. Choice (C), reason, might work if


You advise somebody to do something; you
followed by the preposition for.
advise somebody against doing something.
Choices (A), (B), and (D) are incorrect 133. (A) The adjective is required to describe
prepositions for this idea. the noun advice. Choice (B) is a noun.
Choice (C) is a verb. Choice (D) is the
126. (B) The basic verb form must be used
gerund.
together with to, the shortened form of
in order to, to show the reason for doing 134. (C) Consider means “to think about” and
something. Choice (A) is a noun. Choice fits the idea of this sentence. Choice (A),
(C) is the simple past. Choice (D) is the confuse, means “to mix up” and does not
gerund. make sense in this context. Choice (B),
convince, means “to make somebody
127. (B) The possessive adjective is required.
believe something.” Choice (D), concur,
Choice (A) is the contraction of she is or
means “to agree.”
she has. Choice (C) is the subject pronoun.
Choice (D) is the possessive pronoun. 135. (B) An adjective is needed to describe the
noun director. Choice (A) is a verb or the
128. (A) Expectations means “what you think
noun for this action. Choice (C) is a noun,
will happen” and fits in with the idea of
the person who does this action. Choice
this sentence. Choice (B), experiences,
(D) is the simple past.
means “what has happened to
somebody.” Choice (C), experiments, 136. (B) Attitude means “a state of mind” or
means “trying something to see if it “feeling” and fits the idea of this

INTRODUCTORY COURSE ANSWER KEY: PRACTICE TEST TWO 41

Downloaded by Minh Nguy?n V?n (nguyenvanminh07012003@gmail.com)


lOMoARcPSD|9770843

sentence. Choice (A), assertion, means “a (D) are things you could get at a museum
declaration” or “a statement” and does but do not fit the description.
not make sense in this context. Choice 143. (D) A future verb is required since this
(C), assignment, means “a job or sentence describes something that will
responsibility one needs to do.” Choice happen during the show, which is
(D), attendant, means “somebody who upcoming, that is, in the future. Choice
takes care of something.” (A) is present perfect. Choice (B) is
137. (A) Impossible means “not possible” and simple past. Choice (C) is simple present.
fits the idea of this sentence. Choice (B), 144. (C) This is a recommendation letter, or
impatient, means “not willing to wait” reference. Choice (A) is described in the
and does not make sense in this context. letter but is not the entire purpose of the
Choice (C), improper, means “not correct.” letter. Choices (B) and (D) are other types
Choice (D), impolite, means “rude” or of job-related letters.
“not polite.”
145. (D) The writer is referring to himself
138. (B) The construction so . . . that quantifies here, so the first person possessive
how something was. So is always adjective is needed. Choice (A) is the
followed by an adjective or adverb. second person form. Choices (B) and (C)
Choice (A), too, is a negative idea and are third person forms.
does not make sense in this context.
Choice (C), such, has the same 146. (A) This refers to a new job, which is the
construction as (B), such . . . that, but such subject of the letter. Choices (B), (C), and
is followed by a noun phrase: such a hot (D) are not related to the subject of the
day; such a well-written report. Choice (D), letter.
a lot, must be followed by a comparative 147. (C) A noun follows the article the. Choice
adjective (a lot happier) or a comparative (A) is a present tense verb. Choice (B) is a
adverb (a lot more quickly). past tense verb. Choice (D) is the present
139. (C) A noun is required as the subject of participle.
this sentence. Choice (A) is an adjective. 148. (D) Company employees will use their
Choice (B) is the simple past. Choice (D) own money to pay for travel expenses.
is an adverb. Then they will receive money back, or
Copyright © 2012 by Pearson Education, Inc. Permission granted to reproduce for classroom use.

140. (C) The subordinate conjunction because reimbursement, from the company.
of explains the reason that something is Choices (A), (B), and (C) are not correct
done and fits the idea of this sentence. in this context.
Choice (A), although, means the opposite 149. (C) The department head will approve, or
of what you expect to happen, so it does authorize, expenses that are work-related.
not make sense in this context. Choice Choices (A), (B), and (D) are not correct
(B), in spite of, is incorrect for the same in this context.
reason that (A) is incorrect. Choice (D),
150. (B) The e-mail is about an order that was
consequently, is in the wrong clause.
shipped and hasn’t arrived yet. Choices
(A), (C), and (D) don’t fit the context of
PART 6 (PAGE 281)
the sentence.
141. (A) The announcement is about an art
151. (D) This is a base form verb following the
show, or exhibit. Choices (B), (C), and (D)
modal verb may. Choice (A) uses the
are things that could happen at a
modal will, and two modals cannot be
museum but are not the subject of the
used together. Choice (B) is a gerund.
announcement.
Choice (C) is an infinitive verb.
142. (B) The paragraph discusses something
152. (A) Option is a noun, used as the object of
for which there are prices and which are
the verb prefer. Choice (B) is a verb.
used for admission to the museum, in
Choice (C) is a gerund. Choice (D) is an
other words, tickets. Choices (A), (C), and
adjective.

INTRODUCTORY COURSE ANSWER KEY: PRACTICE TEST TWO 42

Downloaded by Minh Nguy?n V?n (nguyenvanminh07012003@gmail.com)


lOMoARcPSD|9770843

PART 7 (PAGE 285) writing the letter and asking for


feedback. Choice (C) is who should be
153. (B) The job fair offers the opportunity to
called with questions about the form.
meet people currently working in a
Choice (D) is who they would like
variety of fields and who have job
feedback from.
openings. Choice (A) confuses learn to
write a résumé and bring your résumé. 161. (A) The notice serves to let tenants know
Choice (C) confuses similar sounds that service work will be performed on
conference with convention and commerce. the fire alarm system. Choice (B)
Choice (D) confuses buy things on sale confuses an actual fire with service on the
with marketing and commerce. fire alarm system. Choices (C) and (D)
repeat the word fire but are not the
154. (A) The job fair will be held in the
purpose of the notice.
Downtown Convention Center. Choice
(B) is the street by which you can reach 162. (C) If tenants hear the alarm during 8:30
the convention center. Choice (C) is the and 10, they should not be concerned
street on which the convention center is because it is only part of the normal
located. Choice (D) is the sponsor of the service routine. Choice (A) is what
job fair. tenants should do if the alarm goes off at
a time outside of 8:30–10:00. Choice (B) is
155. (D) To make a long distance call, dial 1—
what tenants should do if they have
which is the front desk—to ask for
questions. Choice (D) confuses wait
assistance. Choice (A) is provided as a
patiently and thank you for your patience.
local number of interest. Choice (B)
would be able to connect you with 163. (B) This notice is aimed at homeowners
food, but not be able to connect you to who are thinking about selling their
a long distance number. Choice (C) is homes. Choice (A) is who issued the
provided as a local number of interest. notice. Choice (C) associates researchers
with research and market. Choice (D)
156. (A) Dialing 9 is how you make a local
associates marketing experts with real
call, and 567-555-1014 is the number for
estate market, and confuses similar sounds
City Public Transport. Choice (B) would
experts and experience.
be 9-567-555-2113. Choice (C) would be
Copyright © 2012 by Pearson Education, Inc. Permission granted to reproduce for classroom use.

9-567-555-3456 or 2 for room service. 164. (C) Ms. Ortiz will give a lecture on the
Choice (D) confuses how to make a local real estate market and strategies. Choice
call with dialing 9 to make a local call. (A) confuses estate sale and real estate.
Choice (B) associates a party with
157. (B) To reach maid service, dial 3. Choices
refreshments and event. Choice (D)
(A), (C), and (D) would not connect you
confuses similar sounds competition and
to maid service.
competitive.
158. (A) The letter asks for feedback on Mary
165. (D) You must reserve your space by
Matta’s experience with Technical
calling Mr. Jones if you want to attend.
Support. Choice (B) confuses offering
Choice (A) is why people would want to
support and evaluating the support given.
attend. Choice (B) is incorrect because the
Choice (C) associates selling computers
event is free. Choice (C) is who will speak
with technical support. Choice (D) confuses
at the event.
advertising a website and completing the
customer survey form on the website. 166. (B) These instructions would be found
enclosed in a package with a product that
159. (B) The customer is asked to mail the
was ordered. Choice (A) is where the
completed form or complete it online.
product was purchased from. Choice (C)
Choices (A), (C), and (D) are not correct.
associates post office with mail, package,
160. (B) Ms. Matta, the customer, spoke with return postage, and shipping label. Choice
Joan Kim, a Technical Support (D) associates store with buying, product,
representative. Choice (A) is the person and merchandise.

INTRODUCTORY COURSE ANSWER KEY: PRACTICE TEST TWO 43

Downloaded by Minh Nguy?n V?n (nguyenvanminh07012003@gmail.com)


lOMoARcPSD|9770843

167. (C) The condition of something is the 176. (C) Mr. James will not speak tonight
state of something. The other options do because there is a problem with the
not fit the context. heating system in the auditorium.
Choices (A) and (B) are not mentioned.
168. (A) To return a product within 30 days,
Choice (D) is contradicted by them
repack it in the same box and mail it
having to close the auditorium to repair
back. Choice (B) would not be wise
the heating system.
because after 30 days the refund process
becomes more difficult. Choice (C) is 177. (B) Sharon Rockford will give a talk next
what you should do to return a product Monday. Choice (A) confuses closing at
after 30 days. Choice (D) is not 8:00 P.M. and the talk starting at 8:00 P.M.
mentioned. Choice (C) is not mentioned. Choice (D)
confuses similar sounds résumés (n.) and
169. (C) You will get a full refund, no
resume (v.).
questions asked, if you return a product
within 30 days. Choice (A) is contradicted 178. (D) The World Wide Cafe in Terminal 6
by no questions asked. Choices (B) and (D) provides Internet connection. Choices
are contradicted by you will get a full (A), (B), and (C) do not offer Internet
refund, no questions asked. connection.
170. (C) They are asked to notify the staff in 179. (B) Business Centers do not offer fax
advance of their arrival if they wish to service. Choices (A), (C), and (D) are
make changes in the room arrangements. available.
Choices (A), (B), and (D) are contradicted 180. (C) Taxi stands and bus stops are located
by in advance of your arrival. in the front of each terminal. Choice (A)
171. (B) A complete itinerary will be can only be found in Terminals 1, 4,
distributed when the participants meet and 7. Choice (B) can’t be found in
for dinner. Choice (A) confuses the tour Terminal 5. Choice (D) can only be
schedule with the enclosed airport shuttle found in Terminal 2.
schedule. Choice (C) is when they should 181. (D) The ad lists making appointments
mention they are with the tour so that the among the office assistant’s
staff can inform the tour leader. Choice responsibilities. Choice (A) is confused
Copyright © 2012 by Pearson Education, Inc. Permission granted to reproduce for classroom use.

(D) is unknown. with typing documents. Choice (B) is


172. (D) The itinerary will be printed and sent confused with answering phones. Choice
around to the members of the tour group. (D) is confused with maintaining database.
Choices (A), (B), and (C) do not fit the 182. (C) Ms. Wang’s letter states that she has
context. the required computer skills which,
173. (B) George Harris, the Assistant Director according to the letter, include word
of Tours of World Travel Tours, wrote the processing. Choices (A) and (B) are not
letter. Choices (A), (C), and (D) are mentioned. Choice (D) is confused with
contradicted by Mr. Harris’s title. the fact that the position is in an
architectural firm.
174. (B) Participation in this training seminar is
mandatory for all staff of the Finance 183. (A) Ms. Wang’s letter states I would like to
Office. Choice (A) is who the memo is enter your profession. The letter is
addressed to. Choice (C) is contradicted addressed to Mr. Woo, who, we know
by mandatory for all staff of the Finance from the ad, is an architect. Choice (B)
Office. Choice (D) confuses people contacted confuses high school teacher with call my
by Mr. Oh and contacting Mr. Oh if you high school. Choice (C) is confused with
would like to attend. Ms. Wang’s mention of her night class at
the university. Choice (D) is confused
175. (C) The seminar will take place in
with Ms. Wang’s computer skills.
Conference Room B. Choices (A), (B), and
(D) are not correct.

INTRODUCTORY COURSE ANSWER KEY: PRACTICE TEST TWO 44

Downloaded by Minh Nguy?n V?n (nguyenvanminh07012003@gmail.com)


lOMoARcPSD|9770843

184. (B) In her letter, Ms. Wang states I am 191. (A) Cash is the only form of payment not
taking a night class at the university now. mentioned in the notice. Choices (B), (C),
Choice (A) is confused with Ms. Wang’s and (D) are all acceptable payment
mention of her high school record. options.
Choice (C) is confused with her goal of 192. (A) The office is open Monday through
becoming an architect. Choice (D) is Friday. 555-9754 is the number to call on
confused with her application for the job days when the office is closed. Choice (B)
of office assistant. is the number to call when the office is
185. (A) Ms. Wang states that she has enclosed open. Choices (C) and (D) are Mr.
her résumé. Choice (B) is not mentioned. Wilson’s phone numbers.
Choice (C) is not enclosed; Mr. Woo is 193. (B) In his e-mail, Mr. Wilson says that he
asked to call the high school to obtain it. has to attend a meeting in the afternoon.
Choice (D) is confused with Ms. Wang’s Choice (A) is what Mr. Wilson will not be
mention of the university class she is able to do. Mr. Wilson will not do Choice
taking. (C) because he is using e-mail to make a
186. (D) The party is a farewell for Martha new appointment. Choice (D) is confused
Cunningham, who is moving to another with the assistant sending a check to the
city. Choices (A), (B), and (C) are people dentist’s office.
mentioned in the invitation and e-mail, 194. (B) According to the notice, the
but are not the correct answer. cancellation fee is $40. This is the amount
187. (D) The invitation says that the party is Jim Wilson will pay for his missed
on Thursday, and in his e-mail Tom appointment. Choices (A) and (C) are
apologizes for not attending the party confused with the 24-hours notice
yesterday, so he wrote the e-mail on required to avoid the cancellation fee.
Friday. Choice (A) is the day to reply to Choice (D) is confused with the $40
the invitation. Choice (B) is not cancellation fee.
mentioned. Choice (C) is the day of the 195. (D) Mr. Wilson asks for an evening
party. appointment. From the office hours listed
188. (A) In his e-mail Tom states I had a family on the notice, we know that the Friday is
Copyright © 2012 by Pearson Education, Inc. Permission granted to reproduce for classroom use.

emergency. Choice (B) is confused with the only day of the week that evening
Tom’s statement that he had planned to appointments are available. Choices (A),
attend the party. Choice (C) is confused (B), and (C) are days when only daytime
with the upcoming meeting Tom appointments are available.
mentions at the end of the e-mail. Choice 196. (C) Ms. Lopez is making travel
(D) is confused with the accounting office arrangements for Mr. Pak, so she is a
mentioned in the invitation. travel agent. Choice (A) is not
189. (D) In the invitation, Susan said she mentioned. Choice (B) is incorrect
hoped to raise $300. By Tom’s e-mail, we because Ms. Lopez is arranging airline
know that she was able to raise $75 more flights as well as a hotel room. Choice
than expected, so the total raised was $375. (D) is incorrect because she is arranging a
Choice (A) is the amount Susan asks each hotel room as well as airline flights.
person to contribute. Choice (B) is the 197. (C) According to the first message, 11:15
extra amount of money raised. Choice (C) was the time of Ms. Lopez’s call. Choice
is the amount Susan hoped to raise. (A) is the time she wants Mr. Pak to
190. (C) Tom mentions that he sent a cake to return her call. Choice (B) is the time that
the party. Choice (A) is confused with the Mr. Pak wants to return from his trip.
money raised for the gift. Choice (B) is Choice (D) is the time that Mr. Pak called
confused with Tom’s mention of signing Ms. Lopez.
a card. Choice (D) is confused with the
gift bought for Martha with everyone’s
contributions.

INTRODUCTORY COURSE ANSWER KEY: PRACTICE TEST TWO 45

Downloaded by Minh Nguy?n V?n (nguyenvanminh07012003@gmail.com)


lOMoARcPSD|9770843

198. (D) In his phone message, Mr. Pak says know that Mr. Pak requested the Grand
that he prefers the second option. The Hotel. Choices (B) and (C) are other hotel
second option mentioned in Ms. Lopez’s options mentioned in Ms. Lopez’s
message is Wednesday morning. Choice message. Choice (D) is confused with the
(A) is the time he wants to return from name of Ms. Lopez’s company.
his trip. Choice (B) is the time that there 200. (C) In his message, Mr. Pak says that he
is no flight available. Choice (C) is the has a vacation next month. Choices (A) and
first option mentioned. (B) are confused with times mentioned in
199. (A) In his message, Mr. Pak says that he’ll reference to Mr. Pak’s upcoming business
stay with his first choice, or request for a trip. Choice (D) is not mentioned.
hotel. From Ms. Lopez’s message we
Copyright © 2012 by Pearson Education, Inc. Permission granted to reproduce for classroom use.

INTRODUCTORY COURSE ANSWER KEY: PRACTICE TEST TWO 46

Downloaded by Minh Nguy?n V?n (nguyenvanminh07012003@gmail.com)


lOMoARcPSD|9770843

ANSWER KEY
PRACTICE TEST THREE
PART 1 (PAGE 304) 6. (A) A man is looking at his watch to check
the time. Choice (B) uses the word watch,
1. (D) A train is standing in the station; it
but the man isn’t buying it. Choice (C)
hasn’t left yet. Choice (A) confuses the
confuses the meaning of the word watch.
meaning of the word train—aside from
Choice (D) mentions the man’s jacket, but
being a form of transportation, the word
he is carrying it, not wearing it.
can also be a verb meaning teach. Choice
(B) mentions the passengers and the 7. (D) A woman is at the check-out counter
train, but there are no passengers that of a grocery store, buying fruit and other
appear to be buying tickets. Choice (C) groceries. Choice (A) uses the related
confuses similar-sounding words train word money, but the woman actually has
and rain. a package in her hand. Choice (B)
mentions the cashier’s apron, but
2. (A) A chef is preparing a meal in a
misidentifies its location. Choice (C)
kitchen. Choice (B) confuses similar-
correctly identifies the cashier, but
sounding words kitchen and chicken.
misidentifies his action.
Choice (C) mentions the cook (chef) and
uses the associated word menu, but the 8. (B) This scene of a highway shows a truck
man is cooking, not writing anything. moving along with several cars behind it.
Choice (D) confuses similar-sounding Choice (A) uses the associated word
words cook and book. driver. Choice (C) mentions the cars but
misidentifies their location. Choice (D)
3. (B) Three people are looking at a graph,
mentions the road, but it is not empty
or chart. Choice (A) mentions the pen in
since there is traffic on it.
Copyright © 2012 by Pearson Education, Inc. Permission granted to reproduce for classroom use.

the photo but misidentifies the location.


Choice (C) mentions the table, which has 9. (C) A man is sitting at a table, looking at
several things on it but no food. Choice a computer, and writing on a notepad.
(D) mentions the chart but misidentifies Choice (A) refers to the coffee cup, but it
its location. is sitting on the table; the photo doesn’t
show the man drinking from it. Choice
4. (B) A woman is washing the floor with a
(B) is incorrect because the man is sitting
mop. Choice (A) confuses similar-
at a table, not a desk. Choice (D) is
sounding words floor and door. Choice
incorrect because the man is using a
(C) confuses similar-sounding words
computer, not choosing one.
mop and top. Choice (D) mentions the
floor but is incorrect because there is no 10. (C) The picture shows a man with two
carpet visible. suitcases walking by a bus. Choice (A)
mentions the suitcases but misidentifies
5. (C) The table is set for dinner with plates,
their location. Choice (B) misidentifies
silverware, and china. Choice (A) is
the man’s action – he is walking near or
incorrect because there are no people in
around the bus, not getting on it. Choice
the photo. Choice (B) mentions the
(D) mentions the bus but it is not being
glasses but misidentifies their location.
repaired.
Choice (D) is incorrect because there is no
food in the photo.

INTRODUCTORY COURSE ANSWER KEY: PRACTICE TEST THREE 47

Downloaded by Minh Nguy?n V?n (nguyenvanminh07012003@gmail.com)


lOMoARcPSD|9770843

PART 2 (PAGE 310) 21. (B) This states an opinion about the
restaurant, a logical response to the
11. (C) It’s just down the hall answers where.
question. Choice (A) confuses similar-
Choice (A) repeats the word office. Choice
sounding words restaurant and rest.
(B) confuses similar-sounding words
Choice (C) repeats the word restaurant.
office and officer.
22. (A) This remark about things getting
12. (A) Mary answers the question who.
expensive is a logical response to the
Choice (B) repeats the word desk. Choice
remark about prices going up. Choice (B)
(C) associates package with delivered.
associates prices with cost and fifty dollars.
13. (A) The budget answers the question what. Choice (C) repeats the word recently.
Choice (B) would answer when. Choice
23. (A) Around the corner answers the
(C) confuses similar-sounding words
question where. Choice (B) associates
meeting and reading.
teller with bank. Choice (C) associates
14. (C) Next week answers the question when. money with bank.
Choice (A) repeats the word trip. Choice
24. (C) This explains a reason for preferring a
(B) confuses similar-sounding words
room. Choice (A) confuses similar-
director and directions.
sounding words room and broom. Choice
15. (B) This is an offer to do the favor asked (B) confuses similar-sounding words
for. Choice (A) confuses related words prefer and referred.
package and packed. Choice (C) repeats the
25. (A) They belong to John answers the
word office.
question whose. Choice (B) associates
16. (A) Three or four months answers the garage with car. Choice (C) associates keys
question how long. Choice (B) associates with unlocked.
working with job. Choice (C) confuses
26. (B) In the supply closet answers the
similar-sounding words working and
question where. Choice (A) associates
walking.
photocopier with copies. Choice (C)
17. (B) This is a logical response to the confuses similar-sounding words
suggestion to finish the work after lunch. photocopier and photographer.
Choice (A) gives a reason, confusing the
27. (B) This explains what they were doing.
Copyright © 2012 by Pearson Education, Inc. Permission granted to reproduce for classroom use.

use of the word why in the question—


Choice (A) confuses similar-sounding
which was to make a suggestion, not ask
words late and eight and repeats the
for a reason. Choice (C) confuses similar-
phrase last night. Choice (C) confuses
sounding words work and walk and
similar-sounding words late and plate.
repeats the word lunch.
28. (C) Mrs. Peters answers the question who.
18. (B) This explains the reason for looking
Choice (A) confuses similar-sounding
tired. Choice (A) repeats the word very
words hire and retire. Choice (B) confuses
and confuses similar-sounding words
similar-sounding words hire and tired.
look and book. Choice (C) confuses
similar-sounding words tired and retired. 29. (A) A consulting firm downtown answers
the question where. Choices (B) and (C)
19. (C) Mr. Simon answers the question Who?
repeat coming here.
Choice (A) repeats the word report.
Choice (B) repeats the word meeting. 30. (C) This is a time clause that answers the
question when. Choice (A) confuses
20. (C) The suggestion to bring in more
similar-sounding words dinner and
chairs is a logical response to the remark
thinner. Choice (B) associates dinner and
about the number of people. Choice (A)
hungry.
repeats the word people and confuses
similar-sounding words meeting and 31. (A) On my desk answers the question
greeting. Choice (B) confuses similar- where. Choice (B) associates mail with post
sounding words least and east. office. Choice (C) repeats the word mail.

INTRODUCTORY COURSE ANSWER KEY: PRACTICE TEST THREE 48

Downloaded by Minh Nguy?n V?n (nguyenvanminh07012003@gmail.com)


lOMoARcPSD|9770843

32. (C) This explains the matter, or problem, 43. (D) The man reminds the woman that the
with the fax machine. Choices (A) and (B) meeting is at the conference center. Choice
repeat fax. (A) is where the office is located. Choice (B)
is where they will leave from. Choice (C)
33. (B) This response about messages is a
is mentioned as the place where the
logical response to the question about
meeting will not be held.
phone calls. Choice (A) confuses similar-
sounding words call and hall. Choice (C) 44. (A) The man wants to cash a check, and
confuses similar-sounding words call and he has an account, so he is at a bank.
recall and out and about. Choice (B) associates account with
accountant. Choice (C) associates driver’s
34. (B) Next week answers the question when.
license, the form of identification the man
Choice (A) repeats the word job and
uses, with driving school. Choice (D)
confuses homonyms new and knew.
associates pen with office supply.
Choice (C) repeats the phrase find out.
45. (A) The woman asks to see some form of
35. (A) This answers the question where.
photo ID. Choices (B) and (D) are
Choice (B) confuses homonyms week and
confused with the man asking to cash a
weak. Choice (C) repeats the word week.
check. Choice (C) is what the woman
36. (C) One hour answers the question how gives the man.
long. Choice (A) repeats the word copies.
46. (B) The woman asks the man to sign his
Choice (B) repeats finish.
check, that is, to write his name on it.
37. (A) A cafeteria on the first floor answers the Choice (A) repeats the word license.
question where. Choice (B) repeats the Choice (C) confuses the meaning of the
word lunch. Choice (C) associates lunch word sign. Choice (D) repeats the word
with hungry. cash.
38. (C) This explains a reason, so answers the 47. (C) The man says he will be visiting the
question why. Choice (A) confuses branch office in New York next week.
similar-sounding words yet and jet. Choice (A) confuses out of town with
Choice (B) repeats the words arrived yet. downtown. Choice (B) confuses similar-
39. (B) Three or four answers the question how sounding words branch and ranch. Choice
Copyright © 2012 by Pearson Education, Inc. Permission granted to reproduce for classroom use.

many. Choice (A) repeats the phrase (D) associates accounts with accountant.
applied for the job. Choice (C) uses the 48. (D) This is what the woman finally
related word applications. suggests, and the man agrees. Choice (A)
40. (B) Two years ago answers when. Choice is what the woman first suggests. Choice
(A) repeats the words salary raise. Choice (B) is what the man suggests. Choice (C)
(C) repeats the word salary. is when the woman will be free again.
49. (A) The man says he will check his
PART 3 (PAGE 311) calendar for the next month. Choice (B) is
41. (C) The woman says they can leave at confused with the man’s saying that he
7:30, and the man agrees. Choice (A) is will ask his assistant to call the woman.
when the meeting starts. Choice (B) is Choice (C) confuses the meaning of the
when the woman thought they would word check. Choice (D) is what the man
have to leave. Choice (D) sounds similar will do next week.
to the correct answer. 50. (C) This is when the woman says she will
42. (C) The woman suggests going by have the report ready. Choice (A)
subway and the man agrees. Choices (A) confuses similar-sounding words then
and (B) are what the man says he doesn’t and ten. Choice (B) is when the woman
want to do. Choice (D) confuses similar- will work on the report. Choice (D) is the
sounding words far and car. day of the meeting.

INTRODUCTORY COURSE ANSWER KEY: PRACTICE TEST THREE 49

Downloaded by Minh Nguy?n V?n (nguyenvanminh07012003@gmail.com)


lOMoARcPSD|9770843

51. (A) The man asks the woman to bring the garage. Choice (A) confuses the meaning
report to his office. Choice (B) confuses of the word park. Choice (C) associates
inbox, where the man asks the woman to grocery store (where the garage is near)
put the report, with mailbox. Choice (C) with groceries. Choice (D) associates
repeats the word office. Choice (D) garage with car repair.
associates meeting with conference room. 60. (A) The woman says that the garage is on
52. (B) The woman offers to make the copies the next block. Choice (B) confuses
that the man will need at the meeting. similar sounding words three and fee.
Choice (A) repeats the word meeting. Choices (C) and (D) confuse similar-
Choice (C) repeats the word desk. Choice sounding words while and mile.
(D) repeats the number eight, which is the 61. (A) The woman says that there is no
number of copies the woman suggests charge to use the garage. Choice (B)
making. confuses similar-sounding words for and
53. (C) The man says he will stay three four. Choice (C) confuses similar-
nights. Choice (A) repeats the word one (a sounding words great and eight. Choice
room for one person). Choice (B) confuses (D) confuses similar-sounding words
homonyms two and to. Choice (D) plenty and twenty.
confuses homonyms for and four. 62. (C) The woman says that she is glad for
54. (A) The man says that he will get to the the man. Choice (A) confuses similar-
hotel on Friday. Choices (B) and (C) are sounding words glad and sad. Choice (B)
two of the nights he will be staying at the repeats the word bored, how the man felt
hotel. Choice (D) is the day he will leave. before his promotion. Choice (D)
confuses similar-sounding words hungry
55. (D) The woman asks for the man’s credit
and angry.
card number so that she can make the
reservation. Choice (A) repeats the word 63. (A) The woman says that she wants to
room. Choice (B) confuses similar- celebrate now. Choice (B) is the man’s
sounding words night and flight. Choice suggestion, which the woman doesn’t
(C) repeats the word reservation. agree to. Choice (C) confuses similar-
sounding words then and ten. Choice (D)
56. (C) The woman asks the man about his
Copyright © 2012 by Pearson Education, Inc. Permission granted to reproduce for classroom use.

is when the man will begin his new


flight to Toronto, so he planned to go by
position.
plane. Choices (A) and (D) are mentioned
as not possible. Choice (B) is not 64. (C) The woman says My treat, which
mentioned. means she will pay for the meal. Choice
(A) repeats the word work. Choice (B)
57. (A) Flights have been canceled because of
repeats the word dinner. Choice (D)
the heavy rains. Choice (B) confuses
confuses similar-sounding words treat
similar-sounding words know and snow.
and seat.
Choice (C) confuses similar-sounding
words not and hot. Choice (D) repeats the 65. (B) The woman says that the dinner is for
word clear. some clients. Choice (A) confuses similar-
sounding words seating and meeting.
58. (D) The woman asks when the weather is
Choice (C) confuses the meaning of the
expected to clear up and the man says
word party. Choice (D) confuses similar-
tomorrow. Choice (A) is when the man
sounding words requirement and
planned to leave on his trip. Choice (B)
retirement.
confuses similar-sounding words soon
and afternoon. Choice (C) confuses 66. (B) The woman says that there will be
similar-sounding words flight and sixteen at the dinner. Choices (A), (C), and
tonight. (D) all sound similar to the correct
answer.
59. (B) The man asks for a parking lot and
the woman sends him to a parking

INTRODUCTORY COURSE ANSWER KEY: PRACTICE TEST THREE 50

Downloaded by Minh Nguy?n V?n (nguyenvanminh07012003@gmail.com)


lOMoARcPSD|9770843

67. (C) The woman orders steak. Choice (A) 75. (A) Callers are told to hang up and dial
confuses similar-sounding words nice another number in case of emergency.
and rice. Choice (B) confuses similar- Choice (B) repeats the phrase office hours.
sounding words wish and fish. Choice (D) Choice (C) is how to leave a message for
confuses similar-sounding words kitchen the dentist. Choice (D) is how to make an
and chicken. appointment.
68. (D) The woman says she is getting ready 76. (C) To make an appointment, the caller is
for a job interview. Choice (A) is told to press 2, and then leave some
confused with the trip to the mall information. Choice (A) is not
suggested by the man. Choice (B) mentioned. Choice (B) is what the
associates birthday with party—the assistant will do to help the caller make
woman mentions the suit she received as an appointment. Choice (D) repeats the
a birthday present. Choice (C) repeats the word assistant.
word present. 77. (A) The speaker talks about walking to
69. (D) the woman says she wants black the office and getting up from your desk.
shoes. Choice (A) confuses similar- Choice (B) confuses office with officers.
sounding words like and white. Choice (B) Choice (C) is mentioned as the people
confuses similar-sounding words way who recommend exercise. Choice (D)
and gray. Choice (C) confuses similar- repeats the word gym.
sounding words new and blue. 78. (B) This is the amount recommended by
70. (C) The job interview is on Monday. health experts. Choice (A) sounds like the
Choice (A) repeats the word now. Choice correct answer. Choices (C) and (D) are
(B) confuses similar-sounding words confused with three quarters of an hour.
Monday and Sunday. Choice (D) repeats 79. (D) The speaker recommends exercising
the word month—last month is when she in small amounts throughout the day.
got the suit. Choices (A) and (B) are what the speaker
says is not necessary. Choice (C) repeats
PART 4 (PAGE 314) the word afternoon.
71. (B) The speaker mentions flight
80. (C) It is a store that sells business suits,
attendants, airport, and airlines, so the
Copyright © 2012 by Pearson Education, Inc. Permission granted to reproduce for classroom use.

dresses, sportswear, travel wear, and


announcement is made on a plane.
children’s outfits. Choice (A) is
Choice (A) confuses similar-sounding
associated with refreshments and coffee
words us and bus. Choice (C) repeats the
and snacks. Choice (B) is associated with
word train. Choice (D) is not mentioned.
sportswear. Choice (D) is associated with
72. (B) The speaker says they will arrive in travel wear.
Tokyo in 30 minutes. Choice (A) sounds
81. (D) Everything is marked down 25%.
similar to the correct answer. Choices (C)
Choices (A) and (C) sound similar to the
and (D) confuse our with the similar-
correct answer. Choice (B) is confused
sounding word hour.
with the time the store opens.
73. (D) Passengers are asked to return to
82. (A) The sale is on Saturday and Sunday.
their seats and fasten their seat belts.
Choice (B) is the day the sale ends.
Choice (A) repeats the word schedule.
Choice (C) sounds similar to Sunday.
Choices (B) and (C) are incorrect because
Choice (D) sounds similar to two-day sale.
the speaker tells us that meal and drink
service is now over. 83. (A) The service is called the City Jobs
Corps and helps people find jobs. Choice
74. (C) The office is open Tuesday through
(B) repeats the word computer. Choice (C)
Friday and Saturday. Choices (A), (B),
repeats the word office. Choice (D)
and (D) are incorrect because they don’t
repeats the word hotel.
include all the days that the office is
open.

INTRODUCTORY COURSE ANSWER KEY: PRACTICE TEST THREE 51

Downloaded by Minh Nguy?n V?n (nguyenvanminh07012003@gmail.com)


lOMoARcPSD|9770843

84. (A) The services are offered free of charge. (C) repeat the word ticket. Choice (B)
Choices (B), (C), and (D) sound similar to confuses update and date.
the address given: 35 North Main Street. 94. (A) Train service is indefinitely suspended
85. (C) The speaker says Visit us today. and it is uncertain when it will start again.
Choice (A) is confused with the mention Choice (B) repeats the word evening.
of cover letters. Choice (B) repeats the Choices (C) and (D) are times when buses
word office. Choice (D) is incorrect are scheduled to leave.
because the speaker says that 95. (B) A bridge will be built across the
appointments are not necessary. Wilkus River. Choices (A), (C), and (D)
86. (B) There are high winds, which are are all mentioned but not as something to
expected to continue all night. Choice (A) be constructed.
will be the weather for the next few days. 96. (A) The speaker tells us that construction
Choice (C) confuses similar-sounding will begin in the fall. Choice (B) is not
words train and rain. Choice (D) may be mentioned. Choice (C) is when
the weather later in the week construction will be completed. Choice
87. (C) There have been power outages. Choice (D) confuses similar-sounding words
(A) contradicts what the speaker says: No September and November.
school closings have been reported. Choice 97. (D) The mayor will talk about the bridge
(B) contradicts what the speaker says: during her weekly speech tomorrow.
Trains and subways are running on schedule. Choice (A) repeats the word budget.
Choice (D) is confused with traffic delays Choice (B) is what the mayor did this
near the bridge. afternoon. Choice (C) repeats the word
88. (C) The winds are expected to diminish by museum.
morning. Choices (A) and (B) are when 98. (D) She will speak at the first staff
the windy weather will continue. Choice meeting of next month. Choice (A)
(D) repeats the word week. repeats the word meeting. Choice (B)
89. (C) This is the time the speaker says. repeats the word lunch. Choice (C) is
Choice (A) confuses similar-sounding confused with weekly.
words few and two. Choice (B) sounds 99. (A) She will speak about managing an
Copyright © 2012 by Pearson Education, Inc. Permission granted to reproduce for classroom use.

similar to the correct answer. Choice (D) office. Choice (B) repeats the word
is when the workshops will end. speaking. Choice (C) repeats the word
90. (D) The speaker says Lunch will be served international. Choice (D) repeats the word
in the hotel ballroom on the second floor. videos.
Choices (A) and (C) are where the 100. (C) Listeners are asked to put the date on
registration desk is. Choice (B) is where their calendars. Choice (A) repeats the
the speaker is now. word lunch. Choice (B) confuses the
91. (B) There will be a panel discussion after meaning of the word check. Choice (D)
lunch. Choices (A) and (C) happen in the confuses pressing with press.
morning. Choice (D) repeats the word
registration. PART 5 (PAGE 317)
92. (A) The speaker says that the problem is 101. (C) Both parties will perform the action of
due to flooding on the tracks. Choice (B) signing the document. The subject,
is incorrect because it is the tracks, not document, is not active, so this is a passive
the train, that are damaged. Choice (C) is voice sentence and the past participle
confused with the mention of the seats on verb is required. Choice (A) is base form.
the buses. Choice (D) confuses similar- Choice (B) is present participle. Choice
sounding words seating and heating. (D) is a noun.
93. (D) Buses are being provided to take
passengers to New York. Choices (A) and

INTRODUCTORY COURSE ANSWER KEY: PRACTICE TEST THREE 52

Downloaded by Minh Nguy?n V?n (nguyenvanminh07012003@gmail.com)


lOMoARcPSD|9770843

102. (A) The singular verb, is, agrees with the 114. (C) Inspect means “look at carefully.”
singular subject, person. Choices (B), (C), Choices (A), (B), and (D) look similar to
and (D) are all plural verbs. the correct answer but have meanings
that don’t fit the context.
103. (B) A noun is needed as the subject of the
clause. Choice (A) is a verb. Choice (C) is 115. (A) This is an imperative sentence, giving
a noun but does not have the right a command, so the base form of the verb
meaning for the context. Choice (D) is an is used. Choice (B) is simple present.
adjective. Choice (C) is present participle or
gerund. Choice (D) is future.
104. (B) Resistant means “against.” Choices
(A), (C), and (D) look similar to the 116. (D) The verb object means “oppose” or
correct answer but have meanings that “complain about.” Choices (A), (B), and
don’t fit the context. (C) look similar to the correct answer but
have meanings that don’t fit the context.
105. (A) This is a future real conditional, so a
future tense verb is required in the main 117. (B) This is a future time clause so it needs
clause. Choice (B) is present perfect. a present tense verb. Choice (A) is simple
Choice (C) is simple present. Choice (D) past. Choice (C) is present participle.
is present continuous. Choice (D) is future.
106. (D) This is a preposition of place 118. (C) Since introduces a reason or cause.
describing the position of the supplies on Choices (A), (B), and (D) introduce
top of the shelf. Choices (A), (B) and (C) contradictions.
cannot be correctly used to describe this 119. (B) This completes the infinitive verb
situation. following the main verb plan. Choice (A)
107. (D) A noun is needed as the object of the is an adjective. Choice (C) is a noun.
verb take. Choice (A) is a verb. Choice (B) Choice (D) is an adverb.
is an adjective. Choice (C) is an adverb. 120. (A) As soon as means “after” and
108. (D) This is a past tense unreal introduces the time clause that contains
conditional, requiring would have plus the action that happened first. Choice (B)
past participle in the main clause. Choice means “before.” Choice (C) means “at the
(A) is simple past. Choice (B) is future. same time.” Choice (D) cannot be used to
Copyright © 2012 by Pearson Education, Inc. Permission granted to reproduce for classroom use.

Choice (C) is a present conditional. introduce a time clause.


109. (C) Although introduces a contradiction. 121. (D) This is a present perfect verb used to
Choice (A) is about time. Choices (B) and describe an action that started in the past
(D) introduce a cause or reason. and is still true in the present. Choice (A)
is simple present. Choice (B) is simple
110. (B) This is a preposition of place
past. Choice (C) is present continuous.
describing the location inside the drawer.
Choices (A), (C) and (D) cannot be 122. (C) This is a future real conditional,
correctly used to describe this situation. requiring present tense in the if clause.
Choice (A) is future. Choice (B) is simple
111. (A) Resign means “to quit a job or
past. Choice (D) is past continuous.
position.” Choices (B), (C), and (D) look
similar to the correct answer but have 123. (C) Directive is a noun meaning
meanings that don’t fit the context. “instruction.” Choices (A) and (D) are
verbs. Choice (B) is a noun referring to a
112. (A) The base form of the verb follows a
person.
modal. Choice (B) is past tense. Choice
(C) is present participle. Choice (D) is an 124. (A) Between indicates a position relative
adjective. to two other places or objects. Choices
(B), (C), and (D) are not used to describe
113. (B) Satisfactory is an adjective used to
this situation.
describe the noun job. Choice (A) is a
verb. Choice (C) is a noun. Choice (D) is
an adverb.

INTRODUCTORY COURSE ANSWER KEY: PRACTICE TEST THREE 53

Downloaded by Minh Nguy?n V?n (nguyenvanminh07012003@gmail.com)


lOMoARcPSD|9770843

125. (B) Put off means “postpone.” Choices time clause which describes a concurrent
(A), (C), and (D) can also be combined action. Choice (D) is not used to
with put, but create meanings that don’t introduce a time clause.
fit the context. 137. (C) This is a simple past tense verb
126. (B) Import means “to bring in products describing an action that was completed
from other countries.” Choices (A), (C), last night. Choices (A) and (B) are simple
and (D) look similar to the correct answer present. Choice (D) is infinitive.
but have meanings that don’t fit the 138. (D) This is a passive-voice sentence. The
context. subject, walls, receives the action. Choices
127. (C) The base form of the verb follows a (A), (B), and (C) are all active forms.
modal. Choice (A) is future. Choice (B) is 139. (D) This is an adverb of manner
present participle or gerund. Choice (D) describing the verb speak. Choice (A) is a
is simple past. verb. Choice (B) is an adjective. Choice
128. (C) The main verb suggest is followed by (C) is a noun.
a gerund. Choice (A) is base form. Choice 140. (A) Confuse means “make things difficult
(B) is future. Choice (D) is infinitive. to understand.” Choices (B), (C), and (D)
129. (B) Consent means “permission.” Choices look similar to the correct answer but
(A), (C), and (D) look similar to the have meanings that don’t fit the context.
correct answer but have meanings that
don’t fit the context. PART 6 (PAGE 321)
130. (C) This is a gerund used as the subject of 141. (D) Announced means “to make
the sentence. Choice (A) is base form. information public.” Choices (A), (B), and
Choice (B) is simple present. Choice (D) (C) look similar to the correct answer but
is future. have meanings that don’t fit the context.
131. (A) The adjective different is used with 142. (B) The store will be close to the city’s
the preposition from. Choices (B), (C), and edge, so it will be in the outskirts—the
(D) are prepositions that cannot follow area right outside the city. Choices (A),
the word different. (C), and (D) all refer to areas inside the
city.
Copyright © 2012 by Pearson Education, Inc. Permission granted to reproduce for classroom use.

132. (A) At is a preposition used with hours of


the day. Choices (B), (C), and (D) are 143. (A) Who is a relative pronoun referring to
prepositions that are not used with hours shoppers in this sentence and acting as the
of the day. subject of the adjective clause. Choice (B)
would be in the position of object of the
133. (A) The singular verb has agrees with the
clause. Choice (C) is a possessive form.
singular subject woman. Choices (B), (C),
Choice (D) refers to things, not people.
and (D) are all plural verbs.
144. (A) Although introduces a contradiction:
134. (D) The main verb accept is followed by
It might be expected that registration for
an infinitive verb. Choice (A) is simple
the workshop would not be necessary
present. Choice (B) is simple past. Choice
since no payment is required. Choices (B)
(C) is a gerund.
and (C) introduce a reason. Choice (D)
135. (C) Inside is a preposition describing the cannot be used to introduce a clause.
location of the papers. Choices (A) and
145. (B) Include means “contain” and is often
(B) are adjectives. Choice (D) is a
used to introduce a list. Choices (A), (C),
preposition but is not logical in this
and (D) look similar to the correct answer
sentence.
but have meanings that don’t fit the
136. (B) Before introduces a time clause which context.
describes the second action. Choice (A)
146. (D) Sign up means “register.” Choices (A),
introduces a time clause which describes
(B), and (C) can be used with sign to
the first action. Choice (C) introduces a

INTRODUCTORY COURSE ANSWER KEY: PRACTICE TEST THREE 54

Downloaded by Minh Nguy?n V?n (nguyenvanminh07012003@gmail.com)


lOMoARcPSD|9770843

create phrasal verbs but have meanings courses together. Choice (D) is not
that don’t fit the context. mentioned.
147. (C) The letter is addressed to Mr. Rogers, 157. (B) The dinner was to honor Marcella
who is the owner of, or works for, Inman, who is retiring. Choice (A) is
Tropical Tours. Choices (A), (B), and (D) confused with the mention of Ms.
would mean that the company belongs to Inman’s promotion to branch manager.
the letter writer and/or her husband, but Choice (C) is confused with 35 years of
they are clients of the company. service. Choice (D) is mentioned but was
not the reason for the dinner.
148. (D) This is an adjective used to describe
the tour guide. Choices (A) and (B) are 158. (A) First she was a teller, then she was
verbs. Choice (C) is a noun. promoted to branch manager. Choice (B)
is confused with the location of the
149. (B) Calm means “patient.” Choices (A),
dinner. Choice (C) repeats the word
(C), and (D) are also desirable qualities
teacher, which Ms. Inman used in a
for a tour guide to have but don’t fit the
figurative sense. Choice (D) is associated
meaning of the sentence.
with service.
150. (C) This is the adjective form, describing
159. (A) She has been the bank president for
the noun travel. Choice (A) is a verb.
the past ten years, and now she is
Choices (B) and (D) are nouns.
retiring. Choice (B) is the amount of time
151. (A) This is a third person plural she was a branch manager. Choice (C) is
possessive adjective indicating that the not mentioned. Choice (D) is the total
watches belong to travelers. Choice (B) is amount of time she worked at the bank.
also a possessive adjective but is second
160. (D) Ms. Johnson wrote the letter to talk
person. Choice (C) is a subject or object
about the importance of public
pronoun. Choice (D) is an object
transportation to downtown businesses.
pronoun.
Choice (A) repeats the word business.
152. (D) This is a gerund acting as the subject Choice (B) is incorrect because there is no
of the sentence. Choices (A) and (B) are mention of reelection even though
present tense. Choice (C) is infinitive. Ms. Johnson expresses her approval of
Copyright © 2012 by Pearson Education, Inc. Permission granted to reproduce for classroom use.

the mayor’s position. Choice (C) is


PART 7 (PAGE 325) mentioned but is not the purpose of the
153. (C) OPTIONS courses are offered online letter.
so you can take them using your 161. (D) Commend means “praise.” Choices
computer in your office. Choices (A) and (A), (B), and (C) don’t fit the meaning of
(B) are mentioned as places where you the sentence.
don’t have to go to take the courses.
Choice (D) is the place you call to register 162. (C) The letter talks about businesses
for courses. losing customers because they can’t find
a place to park. Choice (A) is reasonable
154. (A) The word devote means “to give but is not mentioned. Choice (B) repeats
completely for a particular purpose.” the word crowded. Choice (D) is confused
Choices (B), (C), and (D) don’t fit the with the mention of the need to make the
meaning of the sentence. city attractive to businesses.
155. (D) A course goes for one hour a day 163. (A) The City Council will vote on the
Monday–Friday, or 5 hours total. Choice mayor’s proposal next week. Choice (B)
(A) is the number of hours per day. repeats the word month—the business
Choices (B) and (C) are not mentioned. association met last month. Choices (C)
156. (B) One course costs $250. Choice (A) is and (D) repeat the word year—the mayor
the cost per course if you register for two wants to increase the budget next year.
together. Choice (C) is the cost of two

INTRODUCTORY COURSE ANSWER KEY: PRACTICE TEST THREE 55

Downloaded by Minh Nguy?n V?n (nguyenvanminh07012003@gmail.com)


lOMoARcPSD|9770843

164. (B) Rosebud’s cleans clothes, so it is a job is not. Choice (D) is how other hiring
dry-cleaning service. Choice (A) is managers may feel about their jobs.
associated with the clothes. Choice (C) is 173. (D) This is the total amount of all the
associated with cleaning. Choice (D) charges. Choices (A), (B), and (C) are the
repeats the word meeting. amounts charged for different parts of
165. (C) Rates, or prices, can be seen on the the job.
website. Choices (A) and (B) are 174. (C) The invoice is dated May 30 and says
associated with the address and phone that payment is due within fifteen days
number given. Choice (D) is not of that date. Choice (A) is fifteen days
mentioned. before the invoice date. Choice (B) is the
166. (C) The office in advertisement 1 at invoice date. Choice (D) is fifteen days
$1,200 per month has the lowest rent. after payment is due.
Choices (A) and (B) are confused with the 175. (B) DVDs, CDs, and software are the
square footage of two of the offices. items that must be returned with 14 days,
Choice (D) is the rent charged for one of or two weeks, of purchase. Choices (A)
the offices, but it is not the lowest rent. and (D) must be returned within 30 days.
167. (A) The office in advertisement 2 has Choice (C) cannot be returned.
ample parking, and is the only office that 176. (D) This is the correct meaning of the
offers parking. The office in word. Choices (A), (B), and (C) could
advertisement 3 explicitly says that possibly fit the context but do not have
parking is not included. The other two the correct meaning.
advertisements use the word park, but
with a different meaning. 177. (A) There is a 15% charge on items that
are returned after 24 hours of purchase.
168. (A) The ad says to call the realty office for Choices (B) and (C) are confused with the
an appointment. Choices (B), (C), and (D) mention that items in open boxes will be
are all mentioned but not as ways to inspected for damage. Choice (D) repeats
make an appointment. the word receipt.
169. (A) The article is for people preparing for 178. (D) The last line tells us that there are no
a job interview. Choice (B) repeats the time limits on the return of furniture.
Copyright © 2012 by Pearson Education, Inc. Permission granted to reproduce for classroom use.

word résumé. Choice (C) is Ms. Wayman’s Choice (A) applies to snacks and other
job. Choice (D) is mentioned as someone edible items. Choice (B) applies to
who can give advice about interviews. computers and computer peripherals.
170. (C) After receiving résumés, Ms. Wayman Choice (C) applies to most other items.
chooses fifteen candidates to interview. 179. (B) Bus fares will rise 15%. Choice (A) is
Choice (A) is confused with two hundred. confused with 5 years, the last time bus
Choice (B) is the number of top fares increased. Choice (C) is the size of
candidates she has after interviewing. the senior citizen’s discount. Choice (D)
Choice (D) is the number of résumés she is confused with $.30.
receives.
180. (C) The increase will go into effect next
171. (D) Ms. Wayman likes to hire people who month. Choice (A) is when the
show confidence. Choices (A) and (C) are announcement was made. Choice (B) is
mentioned as things people think about confused with the mention of weekend
when preparing for a job interview. fares. Choice (D) is not mentioned.
Choice (B) is not mentioned.
181. (B) Edward Sharpe works as Carolyn
172. (B) Ms. Wayman says that she enjoys her Peters’s office assistant, and she is
job. Choice (A) is used to describe how it writing the letter as a reference for him.
feels to prepare for a job interview. Choice (A) is the person to whom the
Choice (C) is what Ms. Wayman says her letter is addressed, and she works for the
company that is advertising the job

INTRODUCTORY COURSE ANSWER KEY: PRACTICE TEST THREE 56

Downloaded by Minh Nguy?n V?n (nguyenvanminh07012003@gmail.com)


lOMoARcPSD|9770843

opening. Choices (C) and (D) refer to the 188. (C) According to the e-mail, the plans are
company that is advertising the job for Polly, who will report on hiring
opening. procedures, to begin at 10:00. Choices (A)
and (B) will occur before that
182. (A) Edward Sharpe is applying for the
presentation, and choice (D) will occur
advertised job—office manager for Bing
after.
and Rogers engineering firm. Choice (B)
refers to the people who advertised the 189. (A) The information on the agenda says
job. Choice (C) is the person who Edward that lunch will be served in Conference
Sharpe is currently working for. Choice Room A. Choice (B) is where the staff
(D) refers to one of the duties of the meeting will take place. Choice (C) is
advertised job. where Patty’s presentation will take
place. Choice (D) is confused with Liz’s
183. (B) The letter, dated June 30, refers to the
suggestion to Sam to drop by her office
advertisement in yesterday’s newspaper.
after lunch.
Choice (A) is confused with July 10, the
deadline for applying for the job. Choice 190. (D) According to the agenda, Patty’s
(C) is the date of the letter. Choice (D) is presentation, which is on product
the deadline for applying for the job. development, will begin immediately
after lunch, and lunch ends at 1:15.
184. (C) The advertisement gives a post office
Choice (A) is when the meeting begins.
box address for sending in a résumé and
Choice (B) is not mentioned. Choice (C) is
references. Choice (A) repeats the word
when lunch begins.
website, which is mentioned among the
job duties. Choice (B) is what applicants 191. (B) According to the memo, supply
are specifically asked not to do. Choice request forms should be submitted to
(D) is not mentioned. Mr. Wang. Choice (A) is the person who
wrote the memo, Mr. Wang’s boss.
185. (A) The advertisement asks for five years
Choices (C) and (D) repeat the word
experience, and the letter tells us that
assistant.
Edward Sharpe has only two years
experience. Choice (B) is incorrect 192. (B) According to the memo, business card
because the letter tells us that Edward orders take a week to be filled. Choice
Copyright © 2012 by Pearson Education, Inc. Permission granted to reproduce for classroom use.

Sharpe has the software knowledge that (A) is when most office supplies have to
the advertisement asks for. Choice (C) is be ordered. Choices (C) and (D) are
incorrect because the letter of reference confused with the date on the memo.
says many good things about Mr. Sharpe. 193. (D) Orders take two days to be filled.
Choice (D) is incorrect because the letter Sara’s request is dated Wednesday, so she
tells us that Mr. Sharpe is courteous with should have her supplies on Friday.
clients. Choices (A) and (B) are the days
186. (D) According to her e-mail, Liz’s mentioned in the example given in the
meeting with a client Friday morning is memo. Choice (C) is the day the memo
what will make her late for the staff was written.
meeting the same day. Choice (A) is the 194. (C) On the form, Sara requests 3 boxes of
day the e-mail was sent. Choice (B) is not paper clips. Choices (A), (B), and (D) look
mentioned. Choice (C) is the day Liz like quantities entered on the form.
wants to meet with Sam.
195. (D) At the bottom of the form, Sara
187. (A) Liz wrote the e-mail to tell Sam to requests that the supplies be delivered to
give the market report in her place. Room 2B. Choice (A) is Ms. Howland’s
Choice (B) is how it appears on the office number. Choice (B) looks like
agenda. Choices (C) and (D) are people Sara’s office number. Choice (C) is Sara’s
who will present on other topics. office number.

INTRODUCTORY COURSE ANSWER KEY: PRACTICE TEST THREE 57

Downloaded by Minh Nguy?n V?n (nguyenvanminh07012003@gmail.com)


lOMoARcPSD|9770843

196. (A) Ms. Stevens wrote the letter to


complain about the poor quality of the
meals at the hotel. Choice (B) is
mentioned but is not the purpose of the
letter. Choice (C) is associated with the
topic of a hotel. Choice (D) is mentioned
but is not the purpose of the letter.
197. (C) Ms. Stevens writes that she has
stayed at the hotel every summer for the
past seven years. Choices (A), (B), and
(D) are not mentioned.
198. (C) Mr. Grayson writes that the kitchen
was renovated during Ms. Stevens’ stay.
Her letter is dated in July and she
mentions her stay at the hotel last month.
Choices (A) and (B) are when other parts
of the hotel were renovated. Choice (D) is
when Ms. Stevens wrote the letter.
199. (D) Mr. Grayson enclosed a coupon for a
meal at the hotel restaurant. Choice (A) is
associated with the topic of a hotel
complaint but is not mentioned. Choice
(B) repeats the words letter and staff.
Choice (C) is mentioned but is not what
was enclosed.
200. (C) Ms. Stevens normally pays $350 a
night, but on her next visit she will have
the 20% discount offered by Mr. Grayson.
Choice (A) is confused with the size of
Copyright © 2012 by Pearson Education, Inc. Permission granted to reproduce for classroom use.

the discount. Choice (B) is a discount of


$20 rather than 20%. Choice (D) is the
price she normally pays.

INTRODUCTORY COURSE ANSWER KEY: PRACTICE TEST THREE 58

Downloaded by Minh Nguy?n V?n (nguyenvanminh07012003@gmail.com)

You might also like